PDA

View Full Version : Why are Monks considered so bad?



Pages : [1] 2 3

Deaxsa
2013-02-03, 12:50 PM
Basically, I'm just curious. Why are Monks considered so bad? I've always thought of them as being relatively offensively powerful, and defensively strong (in particular with regards to saves).

In particular, a monk with the TWF feat chain and a bit of liberal interpretation of the rules could get... a whooole lot of attacks in a round.

Also, while monks do require only using Charisma or Intelligence as a dump stat, nothing else, the fact that they scale relatively well with 4 stats means that, well, they scale well with 4 stats.

I'm just a bit at a loss as to why they are considered such a poor choice. I've only been on the forums for about a day, but I've already seen a bunch of stuff bashing monks.

flare'90
2013-02-03, 12:53 PM
Could this question wait a few hours?

It's not already Monkday.

limejuicepowder
2013-02-03, 12:54 PM
Please don't start this thread again....is it that time of the week?

Prediction: the next 40 or so posts are going to say the following
-MADness
-low BaB, hit points for a melee'r
-conflicting class abilities (fast movement and flurry)
-much harder to scale damage
-expensive item support

At some point, a noble monk defender will show up and the thread will then become a monk argument for another 10 pages.

Answerer
2013-02-03, 01:03 PM
A fairly thorough of the Monk's problems, what (little) you can do about them, and suggestions for alternatives. (http://rpg.stackexchange.com/a/18970/4563) Doesn't get into most Alternate Class Features; only really notable because that means it doesn't acknowledge Wild Monk. Also, mentioned MAD but doesn't really expound on the point, which is odd because it's one of the biggest problems with the class. But still, goes through every class feature and explains the problems.

Summary:
The best Monk features are all obtained by Monk 2, giving very little reason to stay in the class.
The signature Monk feature, Unarmed Strike, is fairly poor. They're harder to improve than weapon attacks, scale extremely slowly, etc.
Flurry of Blows requires that a Monk not move (bad since he's got low HD and can't wear armor), gives him a penalty (bad since he's got mediocre BAB), and gives him an extra attack (which isn't nearly as good as it could be because he doesn't have bonus damage).
Fast Movement cannot be used at the same time as Flurry of Blows.
The Bonus AC is generally worse than just wearing armor, particularly for a Monk who already needs Str, Dex, and Con.
Almost all of the rest of the Monk's features are really weak, really limited, or both.
Special mention: Slow Fall is honestly insulting.

The only major exceptions are the bonus feats (which are sometimes better than the Fighter's but not usually) and kinda-sorta Evasion.

Elderand
2013-02-03, 01:03 PM
Please don't start this thread again....is it that time of the week?

Prediction: the next 40 or so posts are going to say the following
-MADness
-low BaB, hit points for a melee'r
-conflicting class abilities (fast movement and flurry)
-much harder to scale damage
-expensive item support

At some point, a noble monk defender will show up and the thread will then become a monk argument for another 10 pages.

You forgot class abilities that do the same thing the wizard did but a few level later and do it worse

Eldariel
2013-02-03, 01:17 PM
Check this thread (http://www.giantitp.com/forums/showthread.php?t=269440).

Dusk Eclipse
2013-02-03, 01:20 PM
A fairly thorough of the Monk's problems, what (little) you can do about them, and suggestions for alternatives. (http://rpg.stackexchange.com/a/18970/4563) Doesn't get into most Alternate Class Features; only really notable because that means it doesn't acknowledge Wild Monk. Also, mentioned MAD but doesn't really expound on the point, which is odd because it's one of the biggest problems with the class. But still, goes through every class feature and explains the problems.

Summary:
The best Monk features are all obtained by Monk 2, giving very little reason to stay in the class.
The signature Monk feature, Unarmed Strike, is fairly poor. They're harder to improve than weapon attacks, scale extremely slowly, etc.
Flurry of Blows requires that a Monk not move (bad since he's got low HD and can't wear armor), gives him a penalty (bad since he's got mediocre BAB), and gives him an extra attack (which isn't nearly as good as it could be because he doesn't have bonus damage).
Fast Movement cannot be used at the same time as Flurry of Blows.
The Bonus AC is generally worse than just wearing armor, particularly for a Monk who already needs Str, Dex, and Con.
Almost all of the rest of the Monk's features are really weak, really limited, or both.
Special mention: Slow Fall is honestly insulting.

The only major exceptions are the bonus feats (which are sometimes better than the Fighter's but not usually) and kinda-sorta Evasion.

Wait, you are telling me there are monks that don't take Invisible fist?

Juntao112
2013-02-03, 01:29 PM
In particular, a monk with the TWF feat chain and a bit of liberal interpretation of the rules could miss... a whooole lot of attacks in a round.
Fixed it for you.

Flickerdart
2013-02-03, 01:35 PM
In particular, a monk with the TWF feat chain and a bit of liberal interpretation of the rules could get... a whooole lot of attacks in a round.
-2 from Flurry, -2 from TWF, medium BAB, MADness = all of those attacks miss. And then whatever you were attacking eats your puny d8 hit dice butt alive.

Tvtyrant
2013-02-03, 01:59 PM
-2 from Flurry, -2 from TWF, medium BAB, MADness = all of those attacks miss. And then whatever you were attacking eats your puny d8 hit dice butt alive.

Well there is always the "flurry of bad trip attempts route" using Quick Draw and a bunch of staffs to drop on the tremendous number of failed attempts. Eventually the opponent will roll a one right?

Flickerdart
2013-02-03, 02:04 PM
Well there is always the "flurry of bad trip attempts route" using Quick Draw and a bunch of staffs to drop on the tremendous number of failed attempts. Eventually the opponent will roll a one right?
Rolling a one doesn't mean a failure on an opposed roll. Plus, you can only trip something that's max one size bigger than you, so that strategy is going to become useless very quickly.

silverwolfer
2013-02-03, 02:10 PM
awww no swordsage yet?

White_Drake
2013-02-03, 02:13 PM
We're already ahead of schedule, and ToB isn't for another two days.

MukkTB
2013-02-03, 02:22 PM
Is it Monkday already? You know those anime where some guy is about to fight a bunch of mooks and he says, "You cannot beat me! I know the secret Dragon Melon Golden Rocket fighting style!!!" The mooks ignore him and fight wereupon he owns them. The mooks are monks and that guy is an unarmed swordsage.

Then some big bad comes along. He declares he knows the secret lotus touch, and casts hold person on our hero, rendering him useless.

Flickerdart
2013-02-03, 02:33 PM
Then some big bad comes along. He declares he knows the secret lotus touch, and casts hold person on our hero, rendering him useless.
Fortunately, a Monk 1/Warblade 19 uses Iron Heart Surge to end the Monk character class, and the day is saved.

Popertop
2013-02-03, 02:47 PM
Fortunately, a Monk 1/Warblade 19 uses Iron Heart Surge to end the Monk character class, and the day is saved.

Oh wow, this is gold.


In all seriousness, I like monks.

I just hate how the designers don't.

Like, honestly, by the time you read the ambiguity regarding flurrying while two weapon fighting, it's pretty clear they didn't know what the hell they were doing.

lsfreak
2013-02-03, 02:55 PM
Fortunately, a Monk 1/Warblade 19 uses Iron Heart Surge to end the Monk character class, and the day is saved.

This is amazing.

Something that may be worth pointing out is that Monk 2 tends to be pretty good - because, as already said, monks get pretty much everything they can do by 2nd level. This makes them very good for getting free feats and some other goodies, but only on a chassis that is already good (i.e. not-monk). And, obviously, something that actually benefits from the 2-level dip.

I'll contest that they scale "well" with four stats. Swordsages scale well with Wisdom - it gives them Will saves, AC bonuses, and damage bonuses out-of-the-box, no feats needed. Two-handers scale well with Strength because of it's ramped-up value, both in terms of straight extra damage and the added power attacking you can do with the higher attack bonus. Monks benefit from a lot of stats, but they don't scale well with them - they scale exactly like everyone else with them (except Wisdom). Even taking it as a given that they do scale well with four stats, D&D is a game that rewards specialization and, by extension, punishes jacks-of-all-trades.

Starbuck_II
2013-02-03, 02:57 PM
Oh wow, this is gold.


In all seriousness, I like monks.

I just hate how the designers don't.

Like, honestly, by the time you read the ambiguity regarding flurrying while two weapon fighting, it's pretty clear they didn't know what the hell they were doing.

I hear the 4E Monk isn't bad, we just need someone to convert 4E version to 3.5 and keep the good.

silverwolfer
2013-02-03, 03:11 PM
what about the at will invisibilty monk ACF from one of the evil books


you can now atleast not see the problem now.

GreenSerpent
2013-02-03, 03:26 PM
That's not at-will. You need to spend a Stunning Fist use I think.

I usually for my games port over the Pathfinder Monk and give them the special ability of the Zen Archer archetype instead of Stunning Fist (means they get to roll multiple times on each attack and use the highest for attack, second for crit confirm if it threatens). I also give them a scaling bonus to Jump, Climb, Tumble and Balance (possibly others as well) equal to 4 x Monk level. Boosts them up a fair bit.

Certified
2013-02-03, 03:32 PM
As a fan of the Monk, do people see the same issues with Pathfinder's version?

lsfreak
2013-02-03, 03:32 PM
That's not at-will. You need to spend a Stunning Fist use I think.

Actually it's just a flat once every 3 rounds, as an immediate action. Replaces evasion. Since it's not based on the spell, it doesn't break on damage - only at the end of the round or if dispelled. Easily one of the most useful things you can get out of a Monk 2 dip for a rogue-type.

Dusk Eclipse
2013-02-03, 03:32 PM
Invisible fist is at will every three rounds.

Kalaska'Agathas
2013-02-03, 03:38 PM
As a fan of the Monk, do people see the same issues with Pathfinder's version?

Broadly speaking, yes. There are some interesting options with the Archetypes and ACFs (http://www.d20pfsrd.com/classes/core-classes/monk#TOC-Archetypes-Alternate-Class-Features), but generally speaking the PF Monk doesn't fix the fundamental issues of the 3.5 Monk.

Tvtyrant
2013-02-03, 03:40 PM
Rolling a one doesn't mean a failure on an opposed roll. Plus, you can only trip something that's max one size bigger than you, so that strategy is going to become useless very quickly.

That is what permanent Enlarge Person is for. Besides which, a lot of monsters are medium or large all the way up to CR 20.

Flickerdart
2013-02-03, 03:40 PM
Something that may be worth pointing out is that Monk 2 tends to be pretty good - because, as already said, monks get pretty much everything they can do by 2nd level. This makes them very good for getting free feats and some other goodies, but only on a chassis that is already good (i.e. not-monk). And, obviously, something that actually benefits from the 2-level dip.
I'd argue that Monk may be as long as six levels - Holy Strike is pretty nice, Resistant Body isn't bad, Prayerful Meditation helps against pretty much everything you're likely to fight, and the 6th level bonus from Passive Way is considerable. Of course, you will notice that every single one of these is an ACF, so YMMV.

Dienekes
2013-02-03, 03:42 PM
As a fan of the Monk, do people see the same issues with Pathfinder's version?

Generic monk? Yeah pretty much, I think it might half fix a thing or two but still not very well. Qinggong Monk is supposed to be pretty good.

thethird
2013-02-03, 03:43 PM
It is at will (1 every 3 rounds though)

swordsa monk'd note to self check the thread before posting

Greenish
2013-02-03, 03:43 PM
Since it's not based on the spell, it doesn't break on damage - only at the end of the round or if dispelled.Not at the end of the round. 1 round duration means it ends on the next round at the same initiative count.


Pathfinder monk has some new issues, too, like rulings that Imp. Natural Attack doesn't work with it, or the TWF confusion thingy.

Not to mention that PF Vow of Poverty is both explicitly a monk option (because it apparently wasn't trap enough) and even worse than 3.5 version (ditto).

Doxkid
2013-02-03, 03:50 PM
Monks work wonderfully! For monsters.

It's great for when you want a tough fight with a high power, high HP, hard to kill monster, but don't want that monster using magic to 1shot everyone in the party. It also allows you to quickly boost a monster into an appropriate CR encounter without massive gear.

Flickerdart
2013-02-03, 03:57 PM
high power, high HP, hard to kill monster
Haha no. Monk HD give fewer HP and worse BAB than just taking more of the monster's racial HD. Compare: For every +1 CR, you could have either a Monk level, or 4 HD in most of the types. Even using the crappiest HD (Fey) grants 4d6 HP and +2 BAB compared to the Monk HD's 1d8 HP and +0.75 BAB.

silverwolfer
2013-02-03, 04:29 PM
True, but do they allow you to grab a player and rip them in half with your awesome unarmed damage and grapple rates?

A Troll with monk is a pain if he keeps improved evasion your fireballs

MeiLeTeng
2013-02-03, 04:32 PM
True, but do they allow you to grab a player and rip them in half with your awesome unarmed damage and grapple rates?

A Troll with monk is a pain if he keeps improved evasion your fireballs

Ignoring that Improved Evasion isn't really an issue (and not that useful) why are you trying to drop a Troll with fireballs in the first place?

Also, I don't have any sources in front of me, but I'm pretty sure you're going to find that trying to combine Monk UAS with Natural Attacks isn't going to work like you're hoping it will.

Flickerdart
2013-02-03, 04:39 PM
True, but do they allow you to grab a player and rip them in half with your awesome unarmed damage and grapple rates?
Yes, because more RHD means more size increases which results in bigger grapple bonuses, bigger Strength scores, and bigger natural weapon damage, both of which scale faster than a Monk's crappy unarmed strike. Add to that the extra BAB you're getting from more HD, and it's pretty obvious that Monk levels just don't perform nearly as well.

Beowulf DW
2013-02-03, 04:42 PM
Fortunately, a Monk 1/Warblade 19 uses Iron Heart Surge to end the Monk character class, and the day is saved.

Can I post that in my signature?

BTW, in reference to Pathfinder Monks, a friend just showed me this: http://paizo.com/paizo/blog/v5748dyo5le61?Monkeying-Around

Flickerdart
2013-02-03, 04:52 PM
Can I post that in my signature?
Sure, why not.

As for Pathfinder, I remember there was this whole hullabaloo about using Flurry and TWF where everyone thought it worked together and then SKR came in and said it didn't, and everyone was mad because of the pointless nerf.

Clericzilla
2013-02-03, 04:53 PM
As a fan of the Monk, do people see the same issues with Pathfinder's version?

Please note that PF didn't help melee really at all so ... yeah...

Starbuck_II
2013-02-03, 04:54 PM
True, but do they allow you to grab a player and rip them in half with your awesome unarmed damage and grapple rates?

A Troll with monk is a pain if he keeps improved evasion your fireballs

CR 14 Troll Monk? (remember Trolls are CR 5 and Improved Evasion requires 9 levels)

Pretty high level to worry about Fireballs.

Now CR 7 Trolls (Monk2) with evasion could be a reasonable threat to fireballs.


Please note that PF didn't help melee really at all so ... yeah...

Vitalist is a good multiclass for Monk.

As a Melee Touch (Melee touches can be transfered by Unarmed strikes but at normal not Touch AC) they can stagger enemies or steal heal (both actually).

Hiro Protagonest
2013-02-03, 04:57 PM
Ignoring that Improved Evasion isn't really an issue (and not that useful) why are you trying to drop a Troll with fireballs in the first place?
Fire damage. Of course, Fireball still isn't worth it when fighting one opponent.

Also, I don't have any sources in front of me, but I'm pretty sure you're going to find that trying to combine Monk UAS with Natural Attacks isn't going to work like you're hoping it will.

You can use natural attacks on top of a full attack. Not sure if it works with Flurry.

Answerer
2013-02-03, 04:58 PM
As for Pathfinder, I remember there was this whole hullabaloo about using Flurry and TWF where everyone thought it worked together and then SKR came in and said it didn't, and everyone was mad because of the pointless nerf.
Don't forget that the actual ruling that he made didn't make any kind of sense and no one knew how they were supposed to apply it, because that was a pretty fun aspect of it as well.

My god, SKR should not be able to remain employed in this industry. Didn't Paizo take the hint when Wizards fired him wished him the best in his future endeavors?

Dusk Eclipse
2013-02-03, 04:59 PM
Sadly no, Flurry is a specific full round action different from a full-attack so no, you can't combine Natural Weapons and Flurry.

Answerer
2013-02-03, 05:02 PM
Sadly no, Flurry is a specific full round action different from a full-attack so no, you can't combine Natural Weapons and Flurry.
Incorrect. Flurry of Blows is very explicitly a special modified form of full-attack, and yes, you can add your natural attacks into it. Even better than you otherwise could, even, since a Monk can use not-his-hands for Unarmed Strikes and therefore can still use his claws.

White_Drake
2013-02-03, 05:13 PM
Why would you want to combine flurry with TWF? You would be taking -4 on all of your attacks? I find the idea of somebody building a barbarian/monk with flurry, TWF, snap kick, and whirling frenzy funny. "Check it out guys, I get five attacks!

Answerer
2013-02-03, 06:09 PM
Why would you want to combine flurry with TWF? You would be taking -4 on all of your attacks? I find the idea of somebody building a barbarian/monk with flurry, TWF, snap kick, and whirling frenzy funny. "Check it out guys, I get five attacks!
Yeah, but that's utterly besides the point.

Certified
2013-02-03, 06:09 PM
Can I post that in my signature?

BTW, in reference to Pathfinder Monks, a friend just showed me this: http://paizo.com/paizo/blog/v5748dyo5le61?Monkeying-Around

Disclaimer: I've always liked the style of the Monk and don't really understand the dislike of the class and I'm just now really getting into the heart of Pathfinder. So I put together some numbers based on this Dev post.

With the changes listed there it looks like at 7th Level a Monk can spend 1 Ki point and attack with their Flurry of Blows with a base of +5/+5/+5/+0 treating his attack as Magical, Cold Iron and Silver doing 1d6, or 2d6 if Large (or affected by Enlarge Person). At 8th level this goes up to +6/+6/+6/+1/+1, and 1d10 damage (2d8 Enlarged).

That doesn't count gear, however a +2 Amulet of Mighty Fists is adjusted to 16,000 GP, so feasible by 7th Level, still pricey. A Monk's Robes are 13,000 GP. Also costly, so to move the Monk up to 11th level we can assume they will have both, or possibly even a +3 Amulet.

At 11th Level the Monk strikes as Lawful, Cold Iron, Silver, Magical, using 1 Ki point their base attack is +11/+11/+11/+6/+6/+1 and does 2d8+2 (3d8+2) damage. (Assumes an Amulet +2)

Now the numbers don't factor in ability scores but how under powered compared to comparable level builds?

Greenish
2013-02-03, 06:19 PM
Now the numbers don't factor in ability scores but how under powered compared to comparable level builds?You might want to factor those in, given that one of the big charges leveled against the monk is MAD.

Certified
2013-02-03, 06:25 PM
You might want to factor those in, given that one of the big charges leveled against the monk is MAD.

That's something else I've been meaning to ask about, what is MAD. The only abbreviation I know is Mutually Assured Destruction.

Greenish
2013-02-03, 06:27 PM
That's something else I've been meaning to ask about, what is MAD. The only abbreviation I know is Mutually Assured Destruction.Multiple attribute dependency.

holywhippet
2013-02-03, 06:35 PM
The main problem I have with the monk, which I encountered when I first started playing pen and paper D&D and decided to try a monk as my first character, is the crappy AC of the monk. They might have great saves, special abilities to avoid harm and so forth, but most of the threat a monk (like any character) will face is an enemy attack vs. your AC. The monk just isn't any good in that regard. I'd tried maximising my AC as much as possible but I was still taking damage too often since out opponents were mostly large numbers of goblins and hits that went past my AC were far too often.

In comparison, another player had a dwarf fighter with plate armour and a tower shield. I think the only way the goblins could hurt him in melee was by rolling a 20.

JaronK
2013-02-03, 06:52 PM
That's something else I've been meaning to ask about, what is MAD. The only abbreviation I know is Mutually Assured Destruction.

As someone else mentioned, it's Multiple Attribute Dependency. Some classes don't have this... a Wizard pretty much just needs Int. Dex is nice, Con is nice, but Int is all he really needs. It's even possible to have no attribute dependency, such as an Archivist/Tainted Sorcerer.

Monks need Wis for AC and some of their abilities, Str to actually hurt things, and Dex to stay alive, as well as Con to, well, stay alive some more. That means they can't pump a single primary stat nearly as high, and thus all their save DCs (such as Stunning Fist) and their special attacks (such as tripping) won't be as effective at actually hurting the enemies. It's a serious issue.

JaronK

catsora
2013-02-03, 07:06 PM
Monks are considered weak because they have unsynchronized class features (can't flurry and use fast movement), their hd is only d8 on a frontline class, they can't wear armor as a frontliner, their many attacks will miss most of the time, their later class features can be replicated by spells at earlier levels, they need to have several high stats (painful on point buy:smallfrown:) in order to function. This is only the beginning of the monk's suckiness.

Lupus753
2013-02-03, 07:17 PM
I was incredibly disappointed when I learned of the 3.5 Monk. I was previously familiar with the Monk in Final Fantasy Tactics (where most agree that it's one of the best physical classes). Besides, with the idea of an ascetic using ki and punching everything to death, Monks are my favorite class, going by conception.

I have a good idea as to improving the Monk to usefulness, but it's a near-total rewrite and would probably count as a homebrew. You know it's bad when newbies ask how to optimize the Monk and are just told by veterans to play a Warblade or Ninja or something.

Answerer
2013-02-03, 07:18 PM
(painful on point buy:smallfrown:)
It's honestly painful in general, if you go long enough. Even if you roll four 18s, you cannot (trivially) afford to boost all four of those scores.

Eldariel
2013-02-03, 07:20 PM
Monks need Wis for AC and some of their abilities, Str to actually hurt things, and Dex to stay alive, as well as Con to, well, stay alive some more. That means they can't pump a single primary stat nearly as high, and thus all their save DCs (such as Stunning Fist) and their special attacks (such as tripping) won't be as effective at actually hurting the enemies. It's a serious issue.

To extrapolate: The important distinction for Monk specifically is that the stats all these stats give him are absolutely crucial. Wis only enables him to compensate for being unable to wear armor & Stunning Fist is kinda necessary to get anything at all from the class. Dex likewise. Con compensates for his low HD and is generally required on frontliners. And Str is the only way to compensate for their low attack bonus and to make their only non-sucky special attack, Trip, work.

There are other classes whose mechanics use a lot of stats; Clerics want Strength to hit things, Dex for Initiative, Con for HP/Concentration, Wis for casting and Cha to fuel Turning for instance, but all of it is in secondary mechanics aside from Wis. The key difference here is that a Cleric with 18 Wis and 10 in everything else is going to contribute quite well; sure, he's restricted to a mostly casting build especially early on and he can't fuel turning that well and so on, but he'll be fine since Cleric spell list is good and he still has enough turning to fuel the appropriate feats and his spells can give him big enough buffs to eventually even become a warrior if he wants to. Monk with 18 Str and nothing else? Well, he's going to go squish real fast 'cause on low levels, frontliners are very reliant on AC to survive and this Monk will have 10 AC and 8 HP on level 1 and he still can't meaningfully contribute outside melee combat.

Randomguy
2013-02-03, 07:24 PM
Haha no. Monk HD give fewer HP and worse BAB than just taking more of the monster's racial HD. Compare: For every +1 CR, you could have either a Monk level, or 4 HD in most of the types. Even using the crappiest HD (Fey) grants 4d6 HP and +2 BAB compared to the Monk HD's 1d8 HP and +0.75 BAB.

What about associated/nonassociated class levels? Very few, if any, monsters would have Monk count as an associated class. So that would be 2 monk levels or 4 HD.

2 Monk levels give:

+3 to all 3 saves, which is better than adding RHD thanks to the slightly borked multiclass rules. This helps stop casters wiping out the entire encounter with save or dies.
2 bonus feats and maybe 1 normal feat, while 4 RHD give 1 or 2 normally feats, so monk levels win there.
Evasion, which also helps against the casters.
Wisdom to touch AC, which can be very significant in some monsters, and also help against casters.


Adding a couple of monk levels doesn't make a monster overpoweringly difficult, but it's kind of like putting caltrops under the caster's foot.

Lupus753
2013-02-03, 07:25 PM
Was the Monk any good in previous editions or did it suck from the start? If the former, this might help in homebrews to improve the class.

Answerer
2013-02-03, 07:29 PM
Was the Monk any good in previous editions or did it suck from the start? If the former, this might help in homebrews to improve the class.
Pretty sure it was an afterthought in previous editions.

4e does a better job because the Monk is Psionic.

Flickerdart
2013-02-03, 07:36 PM
What about associated/nonassociated class levels? Very few, if any, monsters would have Monk count as an associated class. So that would be 2 monk levels or 4 HD.

2 Monk levels give:

+3 to all 3 saves, which is better than adding RHD thanks to the slightly borked multiclass rules. This helps stop casters wiping out the entire encounter with save or dies.
2 bonus feats and maybe 1 normal feat, while 4 RHD give 1 or 2 normally feats, so monk levels win there.
Evasion, which also helps against the casters.
Wisdom to touch AC, which can be very significant in some monsters, and also help against casters.


Adding a couple of monk levels doesn't make a monster overpoweringly difficult, but it's kind of like putting caltrops under the caster's foot.
Monk is a melee class, so anything that fights in melee would count it as associated, and anything that doesn't wouldn't want it.

Phelix-Mu
2013-02-03, 07:50 PM
Well, I missed a nice discussion. I need to find that thread I made for "Three Simple Ways to Improve Monk," (http://www.giantitp.com/forums/showpost.php?p=14455112&postcount=1) or w/e I called it...there we go.

I agree monks are terribly flawed. I still like them. Compared to most every other class that is subject to some degree of op, a monk with equal op will not do too well.

And that's why I have my monks multiclass and go around with a party in tow. Even with some fairly minor multiclassing, effectiveness can go way up. My greater mighty wallop monk (cast by an ally, admittedly) can deal sickening amounts of damage, and with a few levels in wiz and abjurant champion, her AC is now almost 60, over sixty with a swift action to cast shield. I do find that moving to charge or otherwise close with the enemy messes up the flurry, but in rounds where a high level monk can land the full greater flurry, massive damage can be dealt.

So, high level monks with support spells and a little multiclassing and PrC aren't hideous, but then you compare them to other optimized, single class melee builds, and the comparison isn't favorable. Still, not sure the monk needs to be able to deal damage at a rate matching the barbarian, since that would rather be stepping on the barbarian's territory a bit.

The biggest problem that I see is that, if you play a monk level by level, BAB is a much bigger issue, the early version of flurry is so funny-bad that you basically shouldn't use it half the time, and balancing where to put money to improve ability scores with items is difficult and expensive.

So, look at my thread above. We actually came up with a decent number of options for DMs that want to homebrew or houserule fixes for monk. My favorite was probably allowing monk to move up to half their speed as a swift action. Fixes many of the flurry issues even early on, without making the silly number of attacks available later on a standard action (which to me spells trouble).

The other fix I am personally considering is allowing monk to pick the best of their Str, Dex, or Wis modifier for their bonus to attack and damage. This fixes a small portion of the MAD, while also saving many monk builds up to a couple feats that are currently crucial (weapon finesse or intuitive attack). Suggestions that monk add Wis to attack and damage, on top of strength, seemed a little aggressive, to me, but perhaps in light of poor damage scaling and difficulty overcoming some forms of DR, it's not as bad as it sounds.

The non-magical disease immunity thing is stupid (save money on flu shots), avoiding *some* falling damage in a small percentage of circumstances is really poorly thought out, SR is not as useful as it sounds, wholeness of body is pathetically small amount even at high levels and impossible to optimize as written, and dimdoor 1/day at poor CL is also not good. What the hell happened to the cool 3.0 Leap of the Clouds ability? WHY DID THEY MAKE 3.5 MONK WORSE THAN 3.0 MONK?

But I still love it. Unarmed swordsage is probably also cool, and clearly more easily op'd and to better effect, and battledancer is also cool if you want a less mystical monk, but I love monk in all it's messy suckage. I love a pet project though, and balancing this class is certainly that.

ACFs, by the way, can salvage some functionality for monk in low to mid-op games. By the time we reach high-op, the votes will fall hard and fast on the side of swordsage. Sad, since it's a non-core, pretty optional resource, and definitely not as simple as core martial classes, making it hard to toss at new players.

Greenish
2013-02-03, 07:52 PM
You know it's bad when newbies ask how to optimize the Monk and are just told by veterans to play a Warblade or Ninja or something.Swordsage (Tome of Battle) is the usual go-to, especially it's unarmed adaption.


What about associated/nonassociated class levels? Very few, if any, monsters would have Monk count as an associated class. So that would be 2 monk levels or 4 HD.Of course, most RHD count for less than unassociated class levels (Aberration, construct, elemental, fey, giant, humanoid, ooze, plant, undead, vermin). Sure, you could give a giant two levels of monk and only increase the CR by 1, but you could also give it +4 RHD for the same CR increase.

Big Fau
2013-02-03, 07:53 PM
At 11th Level the Monk strikes as Lawful, Cold Iron, Silver, Magical, using 1 Ki point their base attack is +11/+11/+11/+6/+6/+1 and does 2d8+2 (3d8+2) damage. (Assumes an Amulet +2)

Now the numbers don't factor in ability scores but how under powered compared to comparable level builds?

Assuming everything hits, and assuming it's 3d8+2 with average damage (6.5 per attack), it's about 117 damage per Full Attack. 3.5 Barbarians can do similar damage with 2 attacks at 11th level.

Phelix-Mu
2013-02-03, 08:05 PM
Assuming everything hits, and assuming it's 3d8+2 with average damage (6.5 per attack), it's about 117 damage per Full Attack. 3.5 Barbarians can do similar damage with 2 attacks at 11th level.

As I did mention above, I don't think damage=similar op barbarian is a good rubric for judging monk. Monks aren't barbarians, and barbarians probably wouldn't like it much if they were. They clearly fill a different party role, though perhaps only slightly so.

As an interesting side note, I'd like to discuss comparing monk effectiveness to the Battledancer from Dragon Magazine Compendium, the rather crazily-written Shaman from OA, and the Kundala from the Book of Erotic Fantasy. All three of these classes have some degree of barehanded combat flavor, but mix in some spellcasting or alternate class abilities, in the case of battledancer.

Answerer
2013-02-03, 08:11 PM
The Battledancer is atrocious. It actually manages to be worse than the Monk.

Kalaska'Agathas
2013-02-03, 08:22 PM
As I did mention above, I don't think damage=similar op barbarian is a good rubric for judging monk.

The trouble is, with the proper feats and items, you can replace Barbarian with Commoner and still achieve the required effect.

Greenish
2013-02-03, 08:32 PM
They clearly fill a different party role, though perhaps only slightly so.What do you feel should be the monk's role? Skills, battlefield control, debuffing, utility?

Lans
2013-02-03, 08:58 PM
-2 from Flurry, -2 from TWF, medium BAB, MADness = all of those attacks miss. And then whatever you were attacking eats your puny d8 hit dice butt alive.
Thats why I think invisible fist almost ups the monk a tier. +2 to hit, and denying the target its dexterity mod is going to be like +4 to hit, and the 50% miss chance means you practically have a d16 hit die.



I'd argue that Monk may be as long as six levels - Holy Strike is pretty nice, Resistant Body isn't bad, Prayerful Meditation helps against pretty much everything you're likely to fight, and the 6th level bonus from Passive Way is considerable. Of course, you will notice that every single one of these is an ACF, so YMMV.
If your going 6 you might as well go 7 for the 50% miss chance as long as your not standing in full daylight.

holywhippet
2013-02-03, 09:14 PM
4e does a better job because the Monk is Psionic.

Psionics have nothing to do with the improvement. It's because 4E have classes which are a lot more balanced because they use the same underlying mechanics.

Phelix-Mu
2013-02-03, 09:15 PM
Probably a mix of scout, mobility, flanker, and status-dealer, at higher levels avoiding damage also adds in an endurance aspect. Stunning fist isn't totally useless, even at low levels. Pain Touch can make it a great tactic on targets that it actually works on, as the enemy is now essentially crippled for two rounds, and drops what they are holding. My monk stunned a balor in the last fight she was in, and it dropped it's vorpal sword and whip.

Monks also can make a decent anti-archer or anti-blaster caster. My epic monk will take this to the logical extreme within a few levels, combining Deflect Arrows, Exceptional Deflection, Reflect Arrows (DM ruled this works on spells that use an attack roll), and Infinite Deflection. Good luck force-orbing her to death.

Clearly monk is not the tank that the barbarian is, so they have to be willing to back out of combat to receive healing (easy with good tumble checks and fast movement) if necessary. If the DM allows sparring dummy of the master item from the 3.0 Arms and Equipment Guide, then the monk can get a 10' step in place of the 5' step that everyone else gets. Only costs a week of training and 30k! With proper support, though, monk doesn't need a particularly huge Constitution, just a willingness to not press their luck in combat. DM understanding and revision of Spring Attack feat tree would help in this respect (in my games, the benefit of Mobility is rolled into Dodge, so Spring Attack is moderately more affordable). Again, you have to settle for being support melee damage dealer.

Kalaska'Agathas
2013-02-03, 09:21 PM
Psionics have nothing to do with the improvement. It's because 4E have classes which are a lot more balanced because they use the same underlying mechanics.

Though, in 3.5, Psionics can definitely improve the Monk. Tashalatora on an Ardent or Psychic Warrior does very well.

Greenish
2013-02-03, 09:22 PM
Probably a mix of scout, mobility, flanker, and status-dealer, at higher levels avoiding damage also adds in an endurance aspect.Scout is sort of a side role, and flanker is class feature, not a role (:smalltongue:). So, you feel monk should be mobile debuffer? That's quite a tough racket to compete in.

Phelix-Mu
2013-02-03, 09:35 PM
Scout is sort of a side role, and flanker is class feature, not a role (:smalltongue:). So, you feel monk should be mobile debuffer? That's quite a tough racket to compete in.

I guess a more narrow definition would be support melee. Flank for the rogue, the tank, or go after archers and spellcasters that are located behind the front lines. A big part of monk looking like a really poor option strategy-wise is that it can't solo melee, and a lot of higher level parties that are caster heavy don't have room for a lot of codependent melee types.

Elderand
2013-02-03, 09:38 PM
If you're entire role can be replaced by your average animal companion/summon, I don't think it speaks well of your utility.

Kalaska'Agathas
2013-02-03, 09:44 PM
That's quite a tough racket to compete in.

What, competing with God (http://www.minmaxboards.com/index.php?topic=1570)?


Monks also can make a decent anti-archer or anti-blaster caster. My epic monk will take this to the logical extreme within a few levels, combining Deflect Arrows, Exceptional Deflection, Reflect Arrows (DM ruled this works on spells that use an attack roll), and Infinite Deflection. Good luck force-orbing her to death.

Deflect Arrows only becomes viable once you've got Infinite Deflection, which means it's only a viable once you've hit epic levels. I say this because dealing damage with arrows, generally speaking, requires firing as many arrows as you can get off in a round. Therefore being able to deflect one arrow doesn't really have much effect. Also, when your DM's house rule isn't in play, force-orbing her to death will be trivial. And even with it in effect, there are so many other options available that not being able to force-orb the target won't be much of a loss.

Flickerdart
2013-02-03, 09:49 PM
If your going 6 you might as well go 7 for the 50% miss chance as long as your not standing in full daylight.
Even if your campaign setting has the Monks of the Dark Moon, in order to belong to the organization, you must give them all of your money, and arguably take a Sorcerer level. So not a great deal.

Greenish
2013-02-03, 09:56 PM
A big part of monk looking like a really poor option strategy-wise is that it can't solo melee, and a lot of higher level parties that are caster heavy don't have room for a lot of codependent melee types.True, I certainly consider the inability to neither take a beating nor to give one a flaw in a melee class. :smallamused:

MeeposFire
2013-02-03, 10:16 PM
Pretty sure it was an afterthought in previous editions.

4e does a better job because the Monk is Psionic.

Well the monk being psionic actually is not new to 4e as even way back in 1e Dragon magazine they said that some of the monks powers were psionic in nature. Further making them psionic did not make them better by default because psionic does not mean much in 4e (any power source does not mean much outside of support and if you mean that then this is a weakness because the best support is for arcane and martial).

Now for the broader question about whether monks were better in other editions well...

1e- Considered weak by most unless you make the high levels and then they are pretty powerful. Hard to get there though and don't be mistaken con is you most important attribute (though only to 16). Evasion is nice though in 1e (which the monk has though it is not called that) since damage effects are more common to be used and are more deadly. Their damage does eventually become decent and the get some nice defensive abilities. Big problem is getting there as living that far is a chore.

Big Plusses

1) eventual power payoff
2) Does have a class role as a thief as it gets the important thief skills of open locks, stealth, and trap finding/removal.

D&D- It is called a "Mystic" in this edition. Offensively becomes vicious and defensively eventually hard to hit. Just like in 1e hard to live at low levels. They also have a major weakness due to lack of owning their own items and being unable to use most basic protection items (bracers, rings, cloaks etc)

4e- Mobile (probably most mobile class in the game) and able to dish out good damage to a lot of enemies. Also lots of fun.


The biggest boost in 4e was the ability to combine mobility and damage in one package and it works well together. The other monks are ok once they get online (usually after a number of levels) and that is too because in those editions they were either offensive powerhouses (D&D what with their large numbers of attacks per round) or mobility and attacking were allowed (1e+2e). 3e double hoses monks by them not being powerhouses (most every warrior class is better than a monk offensively with less investment) and limiting their offense tremendously while trying to be mobile (and mobility is a monk trademark in every edition so it is as if they want to punish you for doing a traditional monk).

On a grand scale looking at past monks what is lacking is

1) Low level survivability. A monk player can make a monk with good AC and decent defenses but not usually at low levels and if they do then they likely killed their offense.

2) Decent offense. You can get that on a monk but it typically will kill you and it still requires a full attack which blows.

3) Too much reliance on full attacks. Mobility is king so why did they build the class around an action that limits it? This is a common problems with 3e warrior classes especially older ones and it is sadly a problem that still is being repeated by a lot of homebrew and heck even 3e clones like PF.

4) No full on class role (3e problem only). Monks have no real role. In 1e they are a thief, 4e a striker, and in D&D a warrior. 3e it is nothing really.

Notice when somebody brings up the swordsage as a replacement it is because it fills these problems. Swordsages have enough defense to not die easily, they can put up good offensive numbers without making themselves too vulnerable, they do not rely on full attacks (strikes are standard actions for the most part so they can run around and still do good damage unlike the monk), and fill the role of a striker (a 4e term but that is what he acts like in 3e where he is about being a damager and in this case a skirmisher type hit and run).

Gigas Breaker
2013-02-03, 10:30 PM
I'm ok with monk just being a dip class. As a two level class it's just fine. Same goes for fighter.

Story
2013-02-03, 11:03 PM
Good luck force-orbing her to death.

Sounds like a job for Hail of Stone. Also, how are you getting True Seeing so you are aware of the attacks and Foresight to not be flatfooted?


Scout is sort of a side role, and flanker is class feature, not a role (:smalltongue:).

It's also a first level spell.

Answerer
2013-02-03, 11:16 PM
It's also a first level spell.
For that matter, it's a first-level maneuver initiated as a swift action.

Snowbluff
2013-02-03, 11:18 PM
I think it is bad.

First, despite numerous changes it still is awful. PF is not a fix. The alternate feature don't fix it. For the actual fix (Swordsage), people complain about us telling people to play it instead.

Second, Monk is like smallpox (http://en.wikipedia.org/wiki/Smallpox), except humanity was able to get rid of smallpox. Monkday is the stray post-eradication outbreaks.

Third, I'd rather be Friar Tuck, not an Asian Monk. Get out of my Western Heroic Fantasy!

You know what, OP? Google it. This is not the first Monkday, nor will it be the last. Maybe this can be less of a waste of energy if you could look back into history and learn of the mistakes of Monkdays past.

Elderand
2013-02-03, 11:37 PM
Third, I'd rather be Friar Tuck, not an Asian Monk. Get out of my Western Heroic Fantasy!

That's hardly a valid point for why monk suck.

TuggyNE
2013-02-03, 11:37 PM
What about associated/nonassociated class levels? Very few, if any, monsters would have Monk count as an associated class. So that would be 2 monk levels or 4 HD.

Most actually would, as mentioned.


2 Monk levels give:

+3 to all 3 saves, which is better than adding RHD thanks to the slightly borked multiclass rules. This helps stop casters wiping out the entire encounter with save or dies.
2 bonus feats and maybe 1 normal feat, while 4 RHD give 1 or 2 normally feats, so monk levels win there.
Evasion, which also helps against the casters.
Wisdom to touch AC, which can be very significant in some monsters, and also help against casters.


Adding a couple of monk levels doesn't make a monster overpoweringly difficult, but it's kind of like putting caltrops under the caster's foot.

The first two levels of Monk are the best; after that it becomes increasingly difficult to justify it. Saying "oh Monk is fine, you can put two levels on a monster and it'll be great!" is kind of seriously missing the point that a base class has 20 levels.

Starbuck_II
2013-02-03, 11:39 PM
Most actually would, as mentioned.



The first two levels of Monk are the best; after that it becomes increasingly difficult to justify it. Saying "oh Monk is fine, you can put two levels on a monster and it'll be great!" is kind of seriously missing the point that a base class has 20 levels.

Monks are still great dip in Pathfinder: Master of Many Styles is awesome.

Snowbluff
2013-02-03, 11:42 PM
That's hardly a valid point for why monk suck.Isn't it? If the crunch sucks and the fluff is misplaced, why is there a monk?

Lupus753
2013-02-03, 11:46 PM
@Snowbluff: Including non-European classes is good for when players want to play that kind of thing. It's too bad that Eastern class concepts like Monk or Samurai turned out so horribly.

Snowbluff
2013-02-03, 11:52 PM
@Snowbluff: Including non-European classes is good for when players want to play that kind of thing. It's too bad that Eastern class concepts like Monk or Samurai turned out so horribly.

Samurai could easily (insufficiently, and yet better) be played as a fighter. Why would they be doing the Monk deal in the Core rules of DnD editions?

Fluff is mutable, but don't give me crap ill-fitting fluff. I can make that up myself, thanks.

Elderand
2013-02-04, 12:01 AM
Samurai could easily (insufficiently, and yet better) be played as a fighter. Why would they be doing the Monk deal in the Core rules of DnD editions?

Fluff is mutable, but don't give me crap ill-fitting fluff. I can make that up myself, thanks.

Your opinion about fluff is just that, an opinion.
You not liking it does not make it bad. It makes it bad for you.
Monks sucks because they are objectively bad. It can be compared mechaanicly to other classes and found wanting. Fluff got nothing to do with it.

Flickerdart
2013-02-04, 12:06 AM
@Snowbluff: Including non-European classes is good for when players want to play that kind of thing. It's too bad that Eastern class concepts like Monk or Samurai turned out so horribly.
I read that as non-Euclidean and for a moment the Monk was a whole lot cooler.

Snowbluff
2013-02-04, 12:09 AM
@^ "Forget logic" is a logical answer. I'd give you an unsalted wheat cracker, but people don't seem to like them.

Your opinion about fluff is just that, an opinion.
You not liking it does not make it bad. It makes it bad for you.
Monks sucks because they are objectively bad. It can be compared mechaanicly to other classes and found wanting. Fluff got nothing to do with it.Stating my opinion is an opinion is not a triumph of logic. We know it's an opinion.


This is a roleplaying game, and everything has a dualistic nature. A class's crunch and it's fluff. When I compared it, I found it wanting. Compared to the other classes, which were designed with what seemed to be medieval western heroic fantasy in mind, while monk was not. The PhB is a carton of eggs, except one of the eggs smells different and makes us ill when we eat it. It's just my opinion the rotten egg tastes bad.

ArcturusV
2013-02-04, 12:25 AM
Bad flavoring is always a valid reason. Even when it comes down to some newer players that I try to get into the game I tend to hear things like:

"Monk... unarmed combat. I can do things like headbutt and piledrive people in combat... cool. But I don't like all that faux shaolin sort of stuff. Can I just be a normal fighter who does stuff like that without all that other stuff?"

Me: Play a fighter, get feats like Improved Unarmed Strike and Improved Grapple. Maybe Power Attack. You'll be good to go.

And all were happy.

Elderand
2013-02-04, 12:35 AM
Bad flavoring is always a valid reason. Even when it comes down to some newer players that I try to get into the game I tend to hear things like:

"Monk... unarmed combat. I can do things like headbutt and piledrive people in combat... cool. But I don't like all that faux shaolin sort of stuff. Can I just be a normal fighter who does stuff like that without all that other stuff?"

Me: Play a fighter, get feats like Improved Unarmed Strike and Improved Grapple. Maybe Power Attack. You'll be good to go.

And all were happy.

Bad flavoring is not a valid reason because what you consider bad flavoring other might enjoy. But bad mechanics and design failure are objective reasons.

Personaly I don't like the flavoring of clerics, does that make it a bad class ? Of course it doesn't.

Fluff is what you use to decide whether you want to play something or not. Whether it fits into your game or not.
Mechanics are what decide whether or not something is efficient and does it's job.
Dislike of fluff apply to one segment of the population, bad mechanics are universal.

It's the same logic behind using RAW to compare classes/builds/rules and not whatever house rule/ban list/abitrary change. Unless of course the point is discussing whether or not to house rule/ban list/random change works.

A topic about why monks are considered bad is best served by discussing the objective flaws of design and mechanics because fluff is mutable and highly subjective.

Pickford
2013-02-04, 12:39 AM
Basically, I'm just curious. Why are Monks considered so bad? I've always thought of them as being relatively offensively powerful, and defensively strong (in particular with regards to saves).

In particular, a monk with the TWF feat chain and a bit of liberal interpretation of the rules could get... a whooole lot of attacks in a round.

Also, while monks do require only using Charisma or Intelligence as a dump stat, nothing else, the fact that they scale relatively well with 4 stats means that, well, they scale well with 4 stats.

I'm just a bit at a loss as to why they are considered such a poor choice. I've only been on the forums for about a day, but I've already seen a bunch of stuff bashing monks.

People hate on monks and like wizards because on a forum you can pretend to be a schroedinger's wizard (i.e. you can always do everything) whereas in actual game play you get 4 spells per level (5 if you're a specialist, but then you can't even cast two spell-schools).

This doesn't just apply to wizards, the claim is also made for Barbarians etc...and generally involves the application of alternative class features and assuming the character is able to easily acquire magic item sets valued upwards of 500k.

Generally speaking anyone claiming any class is amazing is living in an idealized universe where they are conveniently forgetting/house-ruling away all the requirements and downsides of those activities. (Perfect Example: Identify takes an hour to cast and a 100gp pearl...does 'anyone' really play it where you have to do that?)

Another example:
Someone looks at Monk class feature X (let's say, evasion) and says "Oh you could just get a 25k ring and have evasion too! BFD!"

Yeah, but then you're using up one of your two ring slots.

And the ability to reduce damage while falling...yeah it's not as good as feather fall...but who the hell is wasting a slot memorizing feather fall? Slow Fall is good when you consider that nobody in their right mind is blowing a 1st level spell on feather fall at 4th level, let alone 20th.

Incidentally here are the good aspects of pure monk:
Best save progression for all 3. At 20th your base saves are +12. a 9th level spell will have a base DC of 23 (10 + spell level (9) + Int/Cha/Wis (+4 min)) So you have a 45% (end DC 11) of making a save against a 9th level spell, not taking into account your stat modifiers and assuming they have at least a +4 (in order to even cast 9th level spells)

Of course...the monk has Still Mind as well, so if it's an enchantment that's a further +2 dropping the effective DC to 9 (55% chance of passing)

Oh right, but that 20th level monk has a SR of 30. So there's a 50% chance all spells fail outright, 'then' another 50/50 shot at making the save. Cumulatively that's a 25% chance of a spell affecting a monk.

Did I mention that monks can go ethereal (invisible, untouchable) for 20 rounds (need not be consecutive) a day? Sure, they move at half speed...but the speed of a 20th level monk is 90 ft, so you're still moving faster at 45 feet than a non-monk is moving and you're not able to target them unless you blow a true-seeing/see invisibility in one round and then start using force spells. Of course, that just let's you see them as they de-materialize and beat you to death with your own bookbag. (Stunning fist the mage who has an abysmal +6 fort save vs the base DC 20, so it works 70% of the time assuming a wisdom of 10; Or Quivering Palm them as the same save...and since it's more than likely a flurry, you could continue by grappling and pinning the mage which prevents them from speaking (at your option) and making it basically impossible to cast a spell that isn't still/silent metamagicked. (Not a bad reason to have a silent spell teleport prepared always.)

There are alot of possibilities if you check out the various unarmed feats, but it requires hunting through the various supplemental books, it's usually easier for people to find utility spells than the feats that'll make a melee character tick right.

Edit:

Me: Play a fighter, get feats like Improved Unarmed Strike and Improved Grapple. Maybe Power Attack. You'll be good to go.

Oh and you'll do 1d3 damage at 20th whereas a monk would be doing 2d10...but sure.

Snowbluff
2013-02-04, 12:42 AM
@Elderand

I won't call you stupid, because that is not what you are being. You are simply pointing out the incredibly obvious; You are calling me stupid.

This may sound a bit harsh, but you need to realize this is a compounding problem. I gave you a proper comparison of the classes in the PhB, and even a helpful simile. Our understanding of this subject is not flawed. The monk's flavor is from a different setting, and that makes it worse in my eyes. Mutable or not, it is incorrect. The apple is still a kind of fruit, even if you are not in possession of one.

EDIT:
2d10 = 11
1d3 + 8 (Power attack, to compensate for better BaB, regular and greater weapon spec) + 2(Weapon Spec) + 4 (Greater Weapon Spec) = ~18

This is not a huge difference, and is before accounting for fighter's less MAD design (Fighter would have more Str)

ArcturusV
2013-02-04, 12:54 AM
Yeah. Not saying discussing mechanics isn't valid. Just that "because it doesn't fit the flavor" is also valid. Sure we can transmute it. Gods knows I have plenty of times for classes based on setting and player whim. But objectively it's also handy to look at things as they come out of the box. That's how people first read them, that's the mindset they start with. When you first read the monk you are hit with alignment limiting faux shaolin contemplative martial artists who all seem to have taken vows of poverty and such because they don't get to own/use cool stuff, and kinda sticks out as "odd" as much as it did back in Final Fantasy I.

"Okay, so my class choices are a Swordsman in heavy armor. A thief who excels at hitting targets for critical damage. A white mage using healing magic. A black mage using offensive magic. And a Red mage who can do a little healing magic, a little offensive magic, but not the best of both. Oh, and gets to used swords... and then we have an unarmed monk running around in monk clothes, maybe using Nunchucks."

It sticks with some people and it colors how they view the class. *shrug* It might not be wholly material to how it actually works in games. But it happens. Those who tend to LIKE monk flavor in my experience don't want to play DnD. They usually want to play some other system, based off eastern stories, wuxia RPGs, games based off JRPG video games sometimes, or rarely World of Darkness. But DnD tends not to be on their list.

Not that I don't disagree with the "Monks are terrible" ideal mechanically either.

As far as Pickford's "Everyone presumes a Spellcaster will have exactly what he needs ready to go". Well... they SHOULD. Wizards who don't specialize Divination are chumps. It's by far the most powerful school. And when you can literally see into the future or across vast distances whenever you feel like (And only have to ban one school, probably Evocation), you damned well better have exactly the answers you are looking for. Anything else is just pure laziness (And Lazy characters of any class end up subpar). Clerics of course being in a similar boat and also having access to a lot of easy, and powerful, divinations.

I think the killer for Monks though is the fact that they DO need full round actions to do equatable damage to high damage melee fighters. Around a class that can't afford to sit there and trade blows with an enemy. It needs to stay mobile. But it CAN'T in order to use the class features that make it work. It'd be like saying a Bard couldn't use Songs unless he brought a five piece band with him everywhere. It's that painfully limiting to me.

Flickerdart
2013-02-04, 12:58 AM
It'd be like saying a Bard couldn't use Songs unless he brought a five piece band with him everywhere. It's that painfully limiting to me.
You mean your campaigns don't feature adventurer parties of nothing but Bards rocking out across the landscape?

MagnusExultatio
2013-02-04, 12:58 AM
Quivering Palm

http://www.d20srd.org/srd/classes/monk.htm


Quivering Palm (Su)
Starting at 15th level, a monk can set up vibrations within the body of another creature that can thereafter be fatal if the monk so desires. She can use this quivering palm attack once a week, and she must announce her intent before making her attack roll. Constructs, oozes, plants, undead, incorporeal creatures, and creatures immune to critical hits cannot be affected. Otherwise, if the monk strikes successfully and the target takes damage from the blow, the quivering palm attack succeeds. Thereafter the monk can try to slay the victim at any later time, as long as the attempt is made within a number of days equal to her monk level. To make such an attempt, the monk merely wills the target to die (a free action), and unless the target makes a Fortitude saving throw (DC 10 + ½ the monk’s level + the monk’s Wis modifier), it dies. If the saving throw is successful, the target is no longer in danger from that particular quivering palm attack, but it may still be affected by another one at a later time.

http://www.d20srd.org/srd/classes/sorcererWizard.htm#wizard
See spells per day and
http://www.d20srd.org/srd/spells/fingerOfDeath.htm

Greenish
2013-02-04, 12:59 AM
There are alot of possibilities if you check out the various unarmed feats, but it requires hunting through the various supplemental books, it's usually easier for people to find utility spells than the feats that'll make a melee character tick right.Given the various melee combos that float around, I'm going to call that bovine excrement.

Kalaska'Agathas
2013-02-04, 01:01 AM
As far as Pickford's "Everyone presumes a Spellcaster will have exactly what he needs ready to go". Well... they SHOULD. Wizards who don't specialize Divination are chumps.

Even if you don't, it's fairly easy, trivial even, for a Wizard to play at Schrödinger's spell preparation. Spontaneous Divination with Versatile Spellcaster, Alacritous Cogitation, &c.


You mean your campaigns don't feature adventurer parties of nothing but Bards rocking out across the landscape?

With each Bard having a different colored dragon ancestor, for full-laser-light-show-effect Dragonfire Inspiration, natch.

Story
2013-02-04, 01:03 AM
Responses in bold


People hate on monks and like wizards because on a forum you can pretend to be a schroedinger's wizard (i.e. you can always do everything) whereas in actual game play you get 4 spells per level (5 if you're a specialist, but then you can't even cast two spell-schools).

Pretty much any Wizard build could outshine the Monk without even trying. Plus Wizard is a bad example, since a high level Wizard can and will be prepared for anything that doesn't require changing feats or Prcs. And in TO you can change your feats too.

Generally speaking anyone claiming any class is amazing is living in an idealized universe where they are conveniently forgetting/house-ruling away all the requirements and downsides of those activities. (Perfect Example: Identify takes an hour to cast and a 100gp pearl...does 'anyone' really play it where you have to do that?)

Of course not, they get an Artificer's Monocle and never worry about it again.

Another example:
Someone looks at Monk class feature X (let's say, evasion) and says "Oh you could just get a 25k ring and have evasion too! BFD!"

Yeah, but then you're using up one of your two ring slots.

Meanwhile the Monk is using up all their WBL desperately trying to remain relevant.

And the ability to reduce damage while falling...yeah it's not as good as feather fall...but who the hell is wasting a slot memorizing feather fall? Slow Fall is good when you consider that nobody in their right mind is blowing a 1st level spell on feather fall at 4th level, let alone 20th.

No, they just have Heart of Air permanently up. Plus Overland Flight most likely.

Incidentally here are the good aspects of pure monk:
Best save progression for all 3. At 20th your base saves are +12. a 9th level spell will have a base DC of 23 (10 + spell level (9) + Int/Cha/Wis (+4 min)) So you have a 45% (end DC 11) of making a save against a 9th level spell, not taking into account your stat modifiers and assuming they have at least a +4 (in order to even cast 9th level spells)

Of course...the monk has Still Mind as well, so if it's an enchantment that's a further +2 dropping the effective DC to 9 (55% chance of passing)

The Wizard has a way higher int than you'll ever have in any of your stats, so assuming they even decide to allow you a save, you aren't that likely to make it. A 20th level Wizard is going to have at least 34 int. Enjoy your DC 31+s. And who in their right mind would ever use an Enchantment? All the cool kids are immune to mind effecting anyway.

Oh right, but that 20th level monk has a SR of 30. So there's a 50% chance all spells fail outright, 'then' another 50/50 shot at making the save. Cumulatively that's a 25% chance of a spell affecting a monk.

If they have Arcane Mastery, they autopass. Otherwise, they'll just blast you with SR:No spells or take the gamble. Since they probably have a CL of at least 23, That's a 70% chance of beating the SR assuming they didn't bother optimizing at all.

Did I mention that monks can go ethereal (invisible, untouchable) for 20 rounds (need not be consecutive) a day? Sure, they move at half speed...but the speed of a 20th level monk is 90 ft, so you're still moving faster at 45 feet than a non-monk is moving and you're not able to target them unless you blow a true-seeing/see invisibility in one round and then start using force spells. Of course, that just let's you see them as they de-materialize and beat you to death with your own bookbag. (Stunning fist the mage who has an abysmal +6 fort save vs the base DC 20, so it works 70% of the time assuming a wisdom of 10; Or Quivering Palm them as the same save...and since it's more than likely a flurry, you could continue by grappling and pinning the mage which prevents them from speaking (at your option) and making it basically impossible to cast a spell that isn't still/silent metamagicked. (Not a bad reason to have a silent spell teleport prepared always.)

Hey guess what, the 20th level Wizard has ethereal and true seeing permanently if they want it. And you're never even going to get close enough to learn how ineffectual your abilities are.

There are alot of possibilities if you check out the various unarmed feats, but it requires hunting through the various supplemental books, it's usually easier for people to find utility spells than the feats that'll make a melee character tick right.
I think Giacomo's unintentionally proved pretty conclusively that Monk optimization doesn't work.

GenericMook
2013-02-04, 01:09 AM
@Elderand: But fluff mutability does end up being a problem. It's a very valid concern, since DnD is a roleplaying game.

Look at the Fighter: You can be anything from a badass one-man-army, to a traveling warrior-poet. The Fighter's fluff exists completely outside its crunch.

Then look at the Monk: It's almost entirely tied to its fluff. Everything it has tries to tie to the - for a lack of a better term - "faux shaolin" feel. A bar brawler isn't the sort of person to exactly learn how to use nunchucks. The style of Monk characters basically boils down to "'enlightened' unarmed fighter," and doesn't give room for freedom.

Hell, a Barbarian can be anything from a tribal champion to a berserker. It's got way more room for fluff mutability.


People hate on monks and like wizards because on a forum you can pretend to be a schroedinger's wizard (i.e. you can always do everything) whereas in actual game play you get 4 spells per level (5 if you're a specialist, but then you can't even cast two spell-schools).

Scribing spells and/or picking solid spell choices as a wizard can still make you an insane opponent. You can do that in regular play.

This doesn't just apply to wizards, the claim is also made for Barbarians etc...and generally involves the application of alternative class features and assuming the character is able to easily acquire magic item sets valued upwards of 500k.

Except that it's still easy as hell to relatively-optimize a Barbarian, even just factoring feat/ACF/class/PrC choices. Compare that to a Monk, and aside from Tashlatora and a few notable ACFs, your valid options are very limited.

Generally speaking anyone claiming any class is amazing is living in an idealized universe where they are conveniently forgetting/house-ruling away all the requirements and downsides of those activities. (Perfect Example: Identify takes an hour to cast and a 100gp pearl...does 'anyone' really play it where you have to do that?)

Except that Wizards are good because of the sheer variety of spells available to them. I don't know many people that would play Incantatrix at a table, but you can still make a decently optimized Wizard without looking like a giant cheese wheel. You can't do that with a Monk.

Another example:
Someone looks at Monk class feature X (let's say, evasion) and says "Oh you could just get a 25k ring and have evasion too! BFD!"

Yeah, but then you're using up one of your two ring slots.

It's still a class feature that can be bought. If you're playing a game with magic marts, and you can readily buy a ring that gives evasion, why would you even take levels in Monk to do so?

If you aren't playing a Magic Mart world, then that becomes more of a concern, but still isn't significant enough to warrant playing a Monk for Evasion.

And the ability to reduce damage while falling...yeah it's not as good as feather fall...but who the hell is wasting a slot memorizing feather fall? Slow Fall is good when you consider that nobody in their right mind is blowing a 1st level spell on feather fall at 4th level, let alone 20th.

Most people I've played with keep a scroll of Feather Fall for that event.

And seriously, when a Silverbrow Human can get Feather Fall as an SLA at 1st level, it makes a class feature like Slow Fall look like absolute BS.

Incidentally here are the good aspects of pure monk:
Best save progression for all 3. At 20th your base saves are +12. a 9th level spell will have a base DC of 23 (10 + spell level (9) + Int/Cha/Wis (+4 min)) So you have a 45% (end DC 11) of making a save against a 9th level spell, not taking into account your stat modifiers and assuming they have at least a +4 (in order to even cast 9th level spells)

I'll admit that the good saves are decent, but not really worth taking 20 levels in something that doesn't get much more than that.

Of course...the monk has Still Mind as well, so if it's an enchantment that's a further +2 dropping the effective DC to 9 (55% chance of passing)

A +2 is a decent boost, but it's still a pretty crappy class feature that doesn't really scale that well.

Oh right, but that 20th level monk has a SR of 30. So there's a 50% chance all spells fail outright, 'then' another 50/50 shot at making the save. Cumulatively that's a 25% chance of a spell affecting a monk.

SR 30 isn't that spectacular, truth be told. Hell, Clerics get the actual Spell Resistance spell at 9th level, and that's based on Caster Level. It's a decent class feature, but not effective. It's one of those "Comes too late and is trivialized by a lower-level spell anyways" things.

Did I mention that monks can go ethereal (invisible, untouchable) for 20 rounds (need not be consecutive) a day? Sure, they move at half speed...but the speed of a 20th level monk is 90 ft, so you're still moving faster at 45 feet than a non-monk is moving and you're not able to target them unless you blow a true-seeing/see invisibility in one round and then start using force spells. Of course, that just let's you see them as they de-materialize and beat you to death with your own bookbag. (Stunning fist the mage who has an abysmal +6 fort save vs the base DC 20, so it works 70% of the time assuming a wisdom of 10; Or Quivering Palm them as the same save...and since it's more than likely a flurry, you could continue by grappling and pinning the mage which prevents them from speaking (at your option) and making it basically impossible to cast a spell that isn't still/silent metamagicked. (Not a bad reason to have a silent spell teleport prepared always.)

And it's still lackluster. At that point, going ethereal is kinda pointless. It's a waste against generic enemies, and the big bads at that level are morons if they aren't sporting some defense against invisibility.

Also, don't forget that Wizards, Sorcs, Clerics, and especially Druids can boost their Fort save through the roof with Polymorph-brand(TM) cheese.

And then toss buffs on top of that. Yeah.

There are alot of possibilities if you check out the various unarmed feats, but it requires hunting through the various supplemental books, it's usually easier for people to find utility spells than the feats that'll make a melee character tick right.

The problem is that, with a Monk, you're still limited to the shtick of "hit things." And even then, you're not particularly good at it. Flurry misses too easily to be worthwhile.

Edit:


Oh and you'll do 1d3 damage at 20th whereas a monk would be doing 2d10...but sure.

Except that damage dice is kinda trivial when compared to the damage modifiers. If you even remotely optimize a Fighter, that should be no problem at all.

Pickford
2013-02-04, 01:11 AM
EDIT:
2d10 = 11
1d3 + 8 (Power attack, to compensate for better BaB, regular and greater weapon spec) + 2(Weapon Spec) + 4 (Greater Weapon Spec) = ~18

This is not a huge difference, and is before accounting for fighter's less MAD design (Fighter would have more Str)

1) 2d10 (it's +str so you'd be at least 2d10+4) would be 6-24 per hit.

2) Weapon spec is +2, greater weapon spec is +2 for a combined +4.

3) Fighter BAB is only +5 over monk, so if you're just converting that into damage we'd be looking at 1d3 + 5 (pa) + 4 (ws + gws) + 4 (str)


1d3 + 13 or 14-16.

Monk average hit: 15
Fighter average hit: 15

Oh, and the monk gets 2 more hits than the Fighter courtesy of flurry.

Morcleon
2013-02-04, 01:13 AM
...y'know guys... there's a reason why the "Why Each Class Is In Its Tier" thread exists... :smalltongue:

Anyway, monks were (probably) made with the idea for a mobile eastern fighter with some SLAs. And then they made a lot of their best things require full-round actions and being very use limited. This, combined with some things that seemed too good on paper and R&D not having tested stuff out very well, leads to the (lack of) power of the monk.


1) 2d10 (it's +str so you'd be at least 2d10+4) would be 6-24 per hit.

2) Weapon spec is +2, greater weapon spec is +2 for a combined +4.

3) Fighter BAB is only +5 over monk, so if you're just converting that into damage we'd be looking at 1d3 + 5 (pa) + 4 (ws + gws) + 4 (str)


1d3 + 13 or 14-16.

Monk average hit: 15
Fighter average hit: 15

Oh, and the monk gets 2 more hits than the Fighter courtesy of flurry.

2) PA can't be used with unarmed strike.

Get Superior Unarmed Strike. You now have 2d6 damage.

Also, flurry makes you miss far too much.

And why are we having the fighter use unarmed strike anyway? Why not greatsword or something...? :smalltongue:

Story
2013-02-04, 01:15 AM
I just wonder if Pickford has even read any Wizard guides.

4 spells per level, really? I have more than that with my current character and I'm only level 4. Plus scrolls and wands are a thing.

Snowbluff
2013-02-04, 01:15 AM
This is a roleplaying game, and everything has a dualistic nature. A class's crunch and it's fluff. When I compared it, I found it wanting. Compared to the other classes, which were designed with what seemed to be medieval western heroic fantasy in mind, while monk was not. The PhB is a carton of eggs, except one of the eggs smells different and makes us ill when we eat it. It's just my opinion the rotten egg tastes bad.


@Elderand: But fluff mutability does end up being a problem. It's a very valid concern, since DnD is a roleplaying game.
Mooook! *shakes fist in air* (https://www.youtube.com/watch?v=dFlOZFSIsMQ)


*This is getting insipid snip* The fighter's Str is higher. You forgot Weapon Focus, which would add more damage if we were going for similiar hit chance pre-str.

Flickerdart
2013-02-04, 01:19 AM
And why are we having the fighter use unarmed strike anyway? Why not greatsword or something...? :smalltongue:
Because it isn't sporting unless the fighter has both arms tied behind his back, I assume.

Story
2013-02-04, 01:20 AM
That just means the Fighter has to dip Totemist or Warshaper or something for extra Natural Attacks.

Certified
2013-02-04, 01:23 AM
You might want to factor those in, given that one of the big charges leveled against the monk is MAD.

Alright, so I was actually putting together some characters for our next game session so I thought I'd swap out the Fighter I had planned to create for a Monk just to see how things break down. Here is what I came up with:

Frai DuBour
LE Human Monk 7

Str 16 (18)
Dex 17
Con 11
Int 10
Wis 15
Cha 7

HP 38
AC 10 + Dex 3 + Wis 2 + Monk 3 + Dodge 1 + Deflection 1
Ini +7

BAB +5
Movement 50'

Fort +5 +0
Ref +5 +3
Will +5 +2 (+2 bonus on saving throws against enchantment)

Unarmed +10 (2d6+4/x2)
Flurry of Blows +10/+10/+5 (2d6+4/x2)
Dragon Style +10 (2d6+8/x2) +10/+5 (2d6+6/x2)

Class
Evasion
Maneuver Training (Ex): Uses Monk level when calculating his Combat Maneuver Bonus.
Ki Pool (Su): (5)
As long as he has at least 1 point in his ki pool, he can make a ki strike.
Ki Strike: Unarmed Attacks are Treated as magic weapons, cold iron, and silver
Ki Powers Cost: 1 point/Swift Action:
- Make one additional attack at his highest attack bonus with flurry of blows.
- Increase speed by 20 feet for 1 round.
- Gain a +4 dodge bonus to AC for 1 round.
- Gain a +20 bonus on Acrobatics checks made to jump for 1 round.
Ki Powers Cost: 2 points/Swidt Action:
- Wholeness of Body (Su): Heal 14 Hit Points.
Slow Fall (Ex): Takes damage as if the fall were 30 feet shorter than it actually is.
High Jump (Ex): At 5th level, a monk adds his level to all Acrobatics checks made to jump, both for vertical jumps and horizontal jumps. In addition, he always counts as having a running start when making jump checks using Acrobatics.
Purity of Body (Ex): At 5th level, a monk gains immunity to all diseases, including supernatural and magical diseases.

Feats
Improved Initiative (Human)
Dodge (Bonus)
Deflect Arrows (Bonus)
Stunning Fist (Bonus) (7+1) Fort Save 15
Improved Trip (Bonus)
Crushing Blow: You can make a Stunning Fist attempt as a full-round action. If successful, instead of stunning your target, you reduce the target's AC by an amount equal to your Wisdom modifier for 1 minute. This penalty does not stack with other penalties applied due to Crushing Blow.
Dragon Style: While using this style, you gain a +2 bonus on saving throws against sleep effects, paralysis effects, and stunning effects. You ignore difficult terrain when you charge, run, or withdraw. You can also charge through squares that contain allies. Further, you can add 1-1/2 times your Strength bonus on the damage roll for your first unarmed strike on a given round.
Dragon Ferocity: While using Dragon Style, you gain a bonus on unarmed strike damage rolls equal to half your Strength bonus. When you score a critical hit or a successful Stunning Fist attempt against an opponent while using this style, that opponent is also shaken for a number of rounds equal to 1d4 + your Strength bonus.
Ki Throw

Acrobatics (Dex) 7
Climb (Str) 2
Escape Artist (Dex) 7
Knowledge (history) (Int) 2
Knowledge (religion) (Int) 2
Perception (Wis) 7
Sense Motive (Wis) 3
Stealth (Dex) 4
Swim (Str) 1

Equipment 23500
Amulet of Mighty Fists + 1 4,000 gp
Monk's Robes 13,000 GP
Belt of Giant Strength +2 4,000 gp
Ring of Protection +1 2,000 GP

GenericMook
2013-02-04, 01:23 AM
2) PA can't be used with unarmed strike.

Get Superior Unarmed Strike. You now have 2d6 damage.

Also, flurry makes you miss far too much.

And why are we having the fighter use unarmed strike anyway? Why not greatsword or something...? :smalltongue:

Actually, it can. Only thing PA says about Unarmed Strike is this:


You can’t add the bonus from Power Attack to the damage dealt with a light weapon (except with unarmed strikes or natural weapon attacks)

@Snowbluff: Sorry 'bout that. Had already typed it up when I saw your post, and didn't feel like editing it out.

Pickford
2013-02-04, 01:28 AM
As far as Pickford's "Everyone presumes a Spellcaster will have exactly what he needs ready to go". Well... they SHOULD. Wizards who don't specialize Divination are chumps. It's by far the most powerful school. And when you can literally see into the future or across vast distances whenever you feel like (And only have to ban one school, probably Evocation), you damned well better have exactly the answers you are looking for. Anything else is just pure laziness (And Lazy characters of any class end up subpar). Clerics of course being in a similar boat and also having access to a lot of easy, and powerful, divinations.

Except you can't unless your DM is just ignoring spell requirements. You have to be familiar for all the farseeing type spells to operate, otherwise they fizzle, and if you were familiar with the location you wouldn't really need to waste the time trying to scry.

And Contact other plane is risky at best with a minimum DC 7 check for a 34% chance that the entity you contact both knows the correct answer 'and' tells you it.

If you up the chance of getting a correct answer, still capping out at 88% which still makes any response questionable at best, you up the chance you can't cast spells for a week. (i.e. you're totally useless and can't go adventuring in which time conditions may change completely).

Don't get me wrong, I love Divination...but it's not nearly as powerful as you're claiming unless the DM is just throwing you a bone.

Edit:

No, they just have Heart of Air permanently up. Plus Overland Flight most likely.

Except that's not a spell you can use with Permanancy...unless you just mean they took Heart of Air every morning and cast it. In which case they're down to 3 level 2 spells a day and 3 level 5 spells a day.


MagnusExultatio...quivering palm post

The point I was making is that the Monk has a better chance killing a wizard with a quivering palm than a wizard does of killing a monk with finger of touch. And the monk can just layer it in with their other attacks. That's a good thing, not a bad thing. Who cares if it's once a week?

The monk could quivering palm the wizard, go ethereal and hide behind a tree and then just walk away. And will the wizard to die anytime in the next week if the wizard doesn't pay tribute or some-such.


The Wizard has a way higher int than you'll ever have in any of your stats, so assuming they even decide to allow you a save, you aren't that likely to make it. A 20th level Wizard is going to have at least 34 int. Enjoy your DC 31+s. And who in their right mind would ever use an Enchantment? All the cool kids are immune to mind effecting anyway.

Hrm...18 base, +6 max enhancement....+5 points from leveling....so you have 29 int...where'd you get the extra +5? Are you claiming you used wish 5 times in a row for that? Because anything you can do, the monk can also do courtesy of the limitless magic market. So the monk has 34 wisdom too.

Your increased DC is completely negated by the increased monk saves. Either way, this is a stupid argument for you to make as it's not a baseline and 'any' increase to your base DC is counterable by a player, that's why there's no point in futzing around, you'll never gain a leg up in DC over what someone can gain in saves. Heck, I didn't even factor in a cloak of resistance or blur or mirror image, all easily available to the monk.

So no, your monk baseline is going to shrug off a wizard's level 9 spells, baseline, 75% of the time. For lower level spells the percentage goes up. You have no chance of affecting a monk of equal level with a spell below level 3. All things being equal.


If they have Arcane Mastery, they autopass. Otherwise, they'll just blast you with SR:No spells or take the gamble. Since they probably have a CL of at least 23, That's a 70% chance of beating the SR assuming they didn't bother optimizing at all.

I wish my DM would let me have a Schroedinger's wizard too, being able to just change what spells and feats you have loaded out at any given moment would be so convenient.


Hey guess what, the 20th level Wizard has ethereal and true seeing permanently if they want it. And you're never even going to get close enough to learn how ineffectual your abilities are.

You can't use permanancy for true seeing. If you took multiple castings of Etherealness you're giving up a 9th level spell slot for each casting. i.e. you have only 3 more lvl 9 spells that aren't etherealness. And if the monk stops being ethereal in response? You can only play that game for 4 casts at which point you're done.

I probably shouldn't have to even point this out, but everyone talking about item created things would be wise to note that any Wizard who did that would automatically be at least one level behind the curve. Even one less xp is still a level lower. So you'd be lvl 19 at best for dipping into permanancy, in which case you blew 1 of 3 lvl 9 spells on etherealness.


It's still a class feature that can be bought. If you're playing a game with magic marts, and you can readily buy a ring that gives evasion, why would you even take levels in Monk to do so?

If you aren't playing a Magic Mart world, then that becomes more of a concern, but still isn't significant enough to warrant playing a Monk for Evasion.

You're missing the point, you only get 2 ring slots. If you burn a ring slot just to emulate an ability that carries a huge opportunity cost.


Most people I've played with keep a scroll of Feather Fall for that event.

PHB 142: "Many magic items don't need to be activated-magic weapons, magic armor, gauntlets of Dexterity[I], and so forth. However, certain magic items need to be activated, especially potions, scrolls, wands, rods, and staffs. Activating a magic item is a standard action" (side note: "staffs"? heh.)

Since you fall at a rate of 25'/second it would be impossible to activate a scroll of feather fall at a height of under 150' feet. (Because it's a standard action, rather than the actual spell cast, which would be an immediate action).


the big bads at that level are morons if they aren't sporting some defense against invisibility.

Ethereal can't be hit without without a force effect. So even assuming they can see you (DM fiat for example), they still have to find a way to 'hit' you. Etherealness is much better than you're giving it credit for. The monk could easily close the distance to basically anyone in a round or two and the force requirement means they basically have immunity/force.


Also, don't forget that Wizards, Sorcs, Clerics, and especially Druids can boost their Fort save through the roof with Polymorph-brand(TM) cheese.


If you're shape-changing you're not casting spells and you can only shapechange on your turn. Even if they did....the monk doesn't have to activate the palm immediately, he could wait till you shift to something with a lower fort save. Or even just leave and wait till the wizard went to sleep to activate it. You know...when they have no buffs at all.

Flurry doesn't carry any attack penalty past 9th.


The fighter's Str is higher.

Why would that be? Fighter's get no bonus strength and a monk would be stupid if they put less than 18 in str from a 32-point buy.


4 spells per level, really? I have more than that with my current character and I'm only level 4. Plus scrolls and wands are a thing.

You would have to be a specialist wizard to get more and they would have to come from your specialty, so they're not exactly 'open' slots, I was suggesting a non-specialist at any rate. And frankly it really doesn't matter given how poor a chance a wizard has for their spells to work when targeting a monk.

Ah, I see from Tuggyne's post that you're all confused by the terminology I used. I was talking about spell slots, not spells known. Obviously it's possible to get more spells known...although I highly doubt most players actually follow the base rules (100 pages in a spellbook, spells require a page for each level of spell (i.e. 19 pages in your first spellbook are nixed for all the cantrips + 3 1st level spells, so you by default only have 78 pages left for other spells. Let's assume you just account for the basic leveling spells, if you had an 18 int you'd start with 4 more 1st lvl, so that's 74 pages left; At 2nd level it'd be down to 72; At 3rd 68; At 4th 64; at 5th 58; at 6th 52; at 7th 44; 8th 36; 9th 26; 10th 16; 11th 4 pages left wasted. So, on to spellbook #2; 12th 88; 13th 74; 14th 60; 15th 44; 16th 28; 17th 10 pages left, enough for one of the 9th level spells gained at 18th level. On to spellbook #3: 18th (1 spell) to 91; 19th 73; 20th 55;

So in order to have just the basic 60 spells (19 0th; 9 1st; 4 2nd; 4 3rd; 4 4th; 4 5th; 4 6th; 4 7th; 4 8th; 4 9th) that a wizard at 20th would have, you need 3 spell books.

So let's look at your encumberance: That's 9lb's for three books, which doesn't even begin to cover the vaunted "schroedinger's", you actually would only have 4 spell's available at each level, so you might just prepare each once....+5 lbs. for a bedroll, +1lb per day of rations, let's say we're planning for trouble and bring a few extra days (just in cases) beyond a standard 3, bringing us up to 20lbs of gear without even including money...or clothes. Oh then there's the spell pouch +2 lbs, a scholar's outfit +5lbs, and surprise, we're at a medium load for someone with an 8 strength...but hey, at least you're not naked...of course you don't have any money yet, nor are you carry and scrolls, magic items, or weaponry.

So yeah, baseline (i.e. pre-bag of holding that seems mandatory for a caster) you're looking having to pare your spells down drastically to fit in one book. Even post-bag of holding, there are limits on what they can fit/carry.

Tugg: If everyone was playing it straight on the components/mechanics of casting, there would be alot less suggestion that a wizard is overpowered. Most of the powerful spells require expensive (i.e. difficult to acquire/rare) components and the weight of those components would be significant. Heck, just paying for more spells would cost the player a small fortune. Not that most wizards would have the strength to carry all that crap around with them, or necessarily have picked the spells to adequately protect it in their absence.

Many posters in threads on wizards seem to feel that a DM attacking a mage's spellbook is 'cheap'...but a spellbook that's full is worth upwards of 5,000gp...it would be odd for 'nobody' to make attempts on the things. And given that a 20th level wizard would have at least 3 full or near full books, we're talking upwards of 15,000gp of treasure (at least) for robbing/killing a 20th level wizard.

Whereas a monk...carries practically nothing. All their money could be funneled directly into magic items/enhancements. By all rights a wizard would be broke just paying for components, spell books and protection for those two things. When talking about Wealth By Level, many players seem to oh so conveniently forget that.

As for the feather fall thing, I was making the point that a no frills wizard has 4 spell slots at 1st level. Using up one for Feather Fall seems 'very' unlikely to be high on most wizards priorities given so many other obscure options. I don't think it's a stretch to just say that most wizards can easily figure out 4 other spells they would memorize instead and thus not have it. The monk doesn't have to even think about it, they just get it which is kind of nice.

Save don't get weakened at all, worst case scenario you would have at least a +1 to each from stats. Which is still a +13 to your roll (and that's before enhancements to stats from items).

And what is the 'hard counter' from a mage to being grappled? No somatic components, no verbal, you can't use a focus or spell component unless it was already in hand. Plus you have to make a DC 20 + level to cast the spell at all assuming you had a silent teleport memorized. It definitely makes the case for 'doing' that, but all that does is let you escape assuming there's no dimensional anchor/lock on. If there is...good night sally.

I would be curious to see what spells you think a mage is memorizing on a regular basis that they can afford to hold a spell slot for 'just in case' events like that. Enough just in cases and then you can't do anything at all.

Also, who would ever expend quivering palm on an enemy with mirror image up?! Actually a monk with flurry, snap kick and twf could easily hit the10 + dex ac of the images to wipe them all on one round 'and' grapple the mage. Sure, it wouldn't be a pin, but it'd still mess with their casting.

Edit:


Any Wizard smart enough to be able to cast the spell will autopass the checks (remember, you can take 10 on ability checks too).

Besides, Wizards can prepare enough general purpose spells that they usually have something they can do no matter what the situation.

Except you can't take 10 in combat and you can't take 10 on caster level checks. PHB 65 (the only location take 10 and take 20 are described) "Ability Checks and Caster Level Checks: The normal take 10 and take 20 rules apply for ability checks. Neither rule applies to caster level checks (such as when casting [I]dispel magic or attempting to overcome spell resistance."

And even if you 'could' (i.e. emulating skill checks via Arcane Mastery) you wouldn't have the time: "Taking 10: When your character is not being threatened or distracted, you may choose to take 10...Distractions or threats (such as combat) make it impossible for a character to take 10."

So, no there's no auto-pass in combat even with Arcane Mastery.

Edit: Flickerdart

You don't need to take 10; there are many ways of boosting your Intelligence checks high enough that you always make it. A level 20 Wizard can have 36 Int (18 + 2 racial + 6 Headband + 5 levels + 5 inherent) no problem, which is a +13 mod. The rest you can get from Mechanus Mind or Adept Spirit.

The problem is this assumes Schroedinger's Wizard as well, you can't always be a ...whatever race gets a +2 Intelligence...is there an LA with that? Then we're definitely talking non-parity. Might as well give the Monk epic levels if you're assuming the Wizard has 25,000xp worth of inherent and is using a LA race. So the Monk could be 21 or 22 when the Mage is 20...that or the mage wouldn't be 20 and wouldn't have the +5 from levels.

Curmudgeon
2013-02-04, 01:29 AM
2) PA can't be used with unarmed strike.
That's easy enough to check.
You can’t add the bonus from Power Attack to the damage dealt with a light weapon (except with unarmed strikes or natural weapon attacks), even though the penalty on attack rolls still applies. You can use Power Attack with unarmed strikes just fine. The problem is, with medium BAB, you probably can't afford to.

Morcleon
2013-02-04, 01:32 AM
That's easy enough to check. You can use Power Attack with unarmed strikes just fine. The problem is, with medium BAB, you probably can't afford to.

Whoops. :smalltongue: It's still not worth it unless you're TWF, though. :smallbiggrin:

Story
2013-02-04, 01:36 AM
If you up the chance of getting a correct answer, still capping out at 88% which still makes any response questionable at best, you up the chance you can't cast spells for a week. (i.e. you're totally useless and can't go adventuring in which time conditions may change completely).


Any Wizard smart enough to be able to cast the spell will autopass the checks (remember, you can take 10 on ability checks too).

Besides, Wizards can prepare enough general purpose spells that they usually have something they can do no matter what the situation.

Snowbluff
2013-02-04, 01:37 AM
@Snowbluff: Sorry 'bout that. Had already typed it up when I saw your post, and didn't feel like editing it out.It's okay. Worse things have afflicted mankind. Like this thread. lol


That's easy enough to check. You can use Power Attack with unarmed strikes just fine. The problem is, with medium BAB, you probably can't afford to.Yeah, and the fighter would have the advantage for using it.


Whoops. :smalltongue: It's still not worth it unless you're TWF, though. :smallbiggrin:The Fighter's attack is higher. The only output we have to be it the monk's.

TWF won't really work with a Monk. Talk about feat-starving.

TuggyNE
2013-02-04, 01:39 AM
Pickford: you have some good points, but your eagerness to prove all conventional wisdom wrong is overshadowing your judgement a bit; Monks may not be quite as bad as the exaggerations you can sometimes see, but the truth is bad enough. And exaggerating their extremely limited advantages much more wildly is not really going to do you or the class any favors.


People hate on monks and like wizards because on a forum you can pretend to be a schroedinger's wizard (i.e. you can always do everything) whereas in actual game play you get 4 spells per level (5 if you're a specialist, but then you can't even cast two spell-schools).

Plus scrolls, finding other wizards to copy, easy-bake style 6-7 spells/class level, and so on.


Generally speaking anyone claiming any class is amazing is living in an idealized universe where they are conveniently forgetting/house-ruling away all the requirements and downsides of those activities. (Perfect Example: Identify takes an hour to cast and a 100gp pearl...does 'anyone' really play it where you have to do that?)

Since identify isn't really a big thing that proves spellcaster superiority, not sure what the deal is here. And yes, there are in fact people who play without houseruling away caster limitations.

Any time your argument hinges on "I don't believe so-and-so actually plays the way they recommend" you have a bit of a problem, because they may very well do exactly that.


Another example:
Someone looks at Monk class feature X (let's say, evasion) and says "Oh you could just get a 25k ring and have evasion too! BFD!"

Yeah, but then you're using up one of your two ring slots.

I will say that Evasion is one of the more useful features of Monk (and Rogue); the ring is a bit expensive, and not everyone has MIC for combining slots at a moderate surcharge.

Still, Evasion is at level 2, and level 2 is perhaps the best point to drop out of Monk. It doesn't get much better than this.


And the ability to reduce damage while falling...yeah it's not as good as feather fall...but who the hell is wasting a slot memorizing feather fall? Slow Fall is good when you consider that nobody in their right mind is blowing a 1st level spell on feather fall at 4th level, let alone 20th.

"Slow Fall: the awesome class feature that at 20th level almost mimics a first-level spell that is itself so niche no one cares enough about it to ever prepare it." You're uh... not selling the class too great here.


Incidentally here are the good aspects of pure monk:
Best save progression for all 3. At 20th your base saves are +12. a 9th level spell will have a base DC of 23 (10 + spell level (9) + Int/Cha/Wis (+4 min)) So you have a 45% (end DC 11) of making a save against a 9th level spell, not taking into account your stat modifiers and assuming they have at least a +4 (in order to even cast 9th level spells)

Of course...the monk has Still Mind as well, so if it's an enchantment that's a further +2 dropping the effective DC to 9 (55% chance of passing)

Oh right, but that 20th level monk has a SR of 30. So there's a 50% chance all spells fail outright, 'then' another 50/50 shot at making the save. Cumulatively that's a 25% chance of a spell affecting a monk.

Good saves are nice, but they're weakened somewhat (1-2 points, likely, perhaps increasing at later levels) by MADness. Still Mind is handy, but at level 20 you really want immunity to all mind-affecting, not +2 to some.

SR is ... a bit dubious. A fair number of casters will put some effort into overcoming spell resistance with (Greater) Spell Penetration, but those who don't will still often use assay spell resistance and/or true casting, either of which makes SR a lot less interesting, and both of which together make it utterly pointless. Even a caster with nothing but Orange Ioun Stone and Spell Penetration will have a 70% chance of overcoming Monk SR (1d20+23>=30). Further, a sensible caster will pump their casting stat quite a lot beyond +4 at 20th level (seriously? 19 Int/Cha/Wis?), so saves become harder to make, and again: (Greater) Spell Focus is in core, and there are other ways of making saves harder, or even ignoring them entirely. So a ninth-level spell is likely to be more like 10+9+9+1 = 29, not 23.

Of course, none of this necessarily applies to low-op monstrous casters, but then those generally have higher Fort saves than PC races, as well as more HP. Also I think you neglected save boosters entirely, which is more than a little strange, given how important they should be to your argument.

Almost forgot to mention: SR applies to most buffs and cure spells and what-not, and has to be lowered as a standard action. I do hope your party casters know not to try to help you in combat, or else have reliable SR bypassing to ignore your class feature.


Did I mention that monks can go ethereal (invisible, untouchable) for 20 rounds (need not be consecutive) a day? Sure, they move at half speed...but the speed of a 20th level monk is 90 ft, so you're still moving faster at 45 feet than a non-monk is moving and you're not able to target them unless you blow a true-seeing/see invisibility in one round and then start using force spells. Of course, that just let's you see them as they de-materialize and beat you to death with your own bookbag. (Stunning fist the mage who has an abysmal +6 fort save vs the base DC 20, so it works 70% of the time assuming a wisdom of 10; Or Quivering Palm them as the same save...and since it's more than likely a flurry, you could continue by grappling and pinning the mage which prevents them from speaking (at your option) and making it basically impossible to cast a spell that isn't still/silent metamagicked. (Not a bad reason to have a silent spell teleport prepared always.)

To be quite honest, grappling might be the thing Monks were designed for... but unfortunately, as already mentioned, there are excellent and simple hard counters for it. Etherealness can be countered in a number of ways, such as permanent see invisibility, daily true seeing, wall of force/forcecage, and so on; most of these are not especially unusual preparations at 20th level for general use, and some caster builds specialize in using [force] for large amounts of damage.

Your save calculations are a bit off: 6 base + 5 cloak of resistance + 4 from enhanced Con makes a solid 15 (which makes it 80% of the time), and that's assuming you hit the right mirror image and AC, don't trigger celerity, and so on and so forth rather tediously.

Story
2013-02-04, 01:42 AM
Anyway you haven't addressed any of the other points.

I wouldn't be surprised if a level 9 Wizard beat a level 20 Monk with preparation, and the Monk can't do anything to stop the Wizard from preparing.

GenericMook
2013-02-04, 01:45 AM
I wouldn't be surprised if a level 9 Wizard beat a level 20 Monk with preparation, and the Monk can't do anything to stop the Wizard from preparing.

There actually is. It's called distracting the Wizard while he tries to prepare his spells.

Granted, he can always just use whatever leftover spells he has.

Morcleon
2013-02-04, 01:48 AM
There actually is. It's called distracting the Wizard while he tries to prepare his spells.

Granted, he can always just use whatever leftover spells he has.

This assumes that the wizard isn't just sitting in his rope trick box with the rope retracted... :smalltongue:

Flickerdart
2013-02-04, 01:52 AM
Any Wizard smart enough to be able to cast the spell will autopass the checks (remember, you can take 10 on ability checks too).

Besides, Wizards can prepare enough general purpose spells that they usually have something they can do no matter what the situation.
You don't need to take 10; there are many ways of boosting your Intelligence checks high enough that you always make it. A level 20 Wizard can have 36 Int (18 + 2 racial + 6 Headband + 5 levels + 5 inherent) no problem, which is a +13 mod. The rest you can get from Mechanus Mind or Adept Spirit.

zlefin
2013-02-04, 02:20 AM
it's a pity we can't find one standard monk fix that the entire dnd community could agree on; then we could just use that. no need to post that xkcd comic, we've all seen it enough times.

Getting it up to tier 4 is easy; that can be done just by making it a more competent combatant.

Hmm, I should go look up what Legend used for monk imporvements.

Kalaska'Agathas
2013-02-04, 02:23 AM
Except you can't unless your DM is just ignoring spell requirements. You have to be familiar for all the farseeing type spells to operate, otherwise they fizzle, and if you were familiar with the location you wouldn't really need to waste the time trying to scry.

And Contact other plane is risky at best with a minimum DC 7 check for a 34% chance that the entity you contact both knows the correct answer 'and' tells you it.

If you up the chance of getting a correct answer, still capping out at 88% which still makes any response questionable at best, you up the chance you can't cast spells for a week. (i.e. you're totally useless and can't go adventuring in which time conditions may change completely).

Don't get me wrong, I love Divination...but it's not nearly as powerful as you're claiming unless the DM is just throwing you a bone.

May I ask from whence the bolded percentages are derived?

TuggyNE
2013-02-04, 02:28 AM
May I ask from whence the bolded percentages are derived?

Contact other plane, random elemental plane (34% true answer) and greater deity in outer plane (88% true answer).

For a Wizard at level 20, Int should almost certainly be at least +6, and not infrequently at least +10, while even +13 or +14 is not wholly unattainable in practical games. (Also, if there's anything sensible to use rerolls on, it would definitely be COP.)

MeeposFire
2013-02-04, 02:56 AM
it's a pity we can't find one standard monk fix that the entire dnd community could agree on; then we could just use that. no need to post that xkcd comic, we've all seen it enough times.

Getting it up to tier 4 is easy; that can be done just by making it a more competent combatant.

Hmm, I should go look up what Legend used for monk imporvements.

Personally I would say to aim for tier 3 since anymore than that and you would have to give truly ridiculous abilities. With that in mind I would say look at the swordsage. It is tier 3 and works so what does it get that the monk needs? It can attack effectively without full attacks, it has decent AC at all levels, and has special abilities that are useful. So for a fix you could start with...

1. Flurry can be made to be used like the snap kick feat. This allows for mobility to work indeed it can even make spring attack effective if you make it part of an attack action. With all the optimization out there for unarmed attacks you could do effective damage without relying on charging or full attack actions.

2. Give the full monk AC bonus at level one or something close to it. It does not need to even improve because high level monk AC will be comparable to most characters it is the low levels that need the boost and getting a base AC that is equal to a swordsage would help a lot.

3. IN addition to keeping most or all the old abilities that you like add a few abilities (I like making them encounter based). For example at level 20 you could give the ability to do two rounds worth of actions in one round (aka Island of Time in ToB). Encounter based teleports at a fair action cost would work as well. These sort of abilities would give you flexibility in an encounter.

This is just a start not a final answer of course. Another thing that needs to be done is to either make available some way of using enchanted unarmed strikes easily/cheaply (RAW ways include heavy arms graft and a battlefist), homebrew something for that, or allow gauntlets to work with flurry (swordsages can deal their full damage with gauntlets and lose nothing where monks lose flurry).

Greenish
2013-02-04, 03:04 AM
it's a pity we can't find one standard monk fix that the entire dnd community could agree on; then we could just use that.If people would just stop protesting against swordsage… :smalltongue:


Hmm, I should go look up what Legend used for monk imporvements.Full BAB, Wis to attack/damage/combat maneuvers, ability to wear armour, enchantable unarmed strike, standard action full attack, flight, fly-by attack, teleport, rebirth, loads of debuffs, immunities (including one round immunity to damage)…

You know, the usual.

Snowbluff
2013-02-04, 03:05 AM
If people would just stop protesting against swordsage… :smalltongue:

That and Tashalatora Psywar.:smalltongue:

Flickerdart
2013-02-04, 03:32 AM
Contact other plane, random elemental plane (34% true answer) and greater deity in outer plane (88% true answer).
Standard practice is to double-check.

Juntao112
2013-02-04, 03:33 AM
I think that I have found the perfect monk fix. (http://www.enworld.org/forum/showthread.php?303807-The-Hao-Han)

Morcleon
2013-02-04, 03:36 AM
I think that I have found the perfect monk fix. (http://www.enworld.org/forum/showthread.php?303807-The-Hao-Han)

It still doesn't solve the problem of MAD or lack of stuff. In fact, it may actually be worse than the original monk (not having any sort of Su abilities at all, nor Wis to AC nor evasion).

You could probably give the monk all of it's things at will, and it still wouldn't be particularly better... :smalltongue:

Juntao112
2013-02-04, 03:39 AM
(Its a punchbarian.)

Greenish
2013-02-04, 03:39 AM
It still doesn't solve the problem of MAD or lack of stuff. In fact, it may actually be worse than the original monk (not having any sort of Su abilities at all, nor Wis to AC nor evasion).I'm fairly certain that barbarian is generally agreed to be a better class than the monk.

Morcleon
2013-02-04, 03:40 AM
(It's a punchbarian.)

Where's the punch part? It doesn't even get unarmed strike stuff... :smalltongue:


I'm fairly certain that barbarian is generally agreed to be a better class than the monk.

It is, but then it's just a separate class, not a fix. :smallwink:

Juntao112
2013-02-04, 03:43 AM
Where's the punch part? It doesn't even get unarmed strike stuff...
Pay very close attention to the ACFs.

The Monk does get a few ACFs; Invisible Fist that is legitimately good, for instance. And it has a few tricks here and there; Barbarians are focused mainly on beating things up while Monks have occasionally gotten something for action economy or utility.

But overall, if you want to beat something to death, Barbarian is probably a better choice.

Just curious, but is there a way to get a monk of any race, but especially a dwarf, the outsider subtype without too much LA?

Kelb_Panthera
2013-02-04, 06:55 AM
CoP is very limited in its ability to discern the future. Only greater deities can actually see the future and then only in regards to their portfolios. They're also limited in just how far they can see, albeit a fair distance; 1 week per divine rank to a minimum of 16 weeks and a maximum of 20 depending on the deity.

This translates to needing to know which god to ask and hoping that the GM doesn't just block you outright for trying to circumvent his challenges (CoP has a clause in its text that deities sometimes block it.) Asking multiple times doesn't really help unless there's more than one god that covers the field your question is in.

Pickford
2013-02-04, 08:32 AM
Standard practice is to double-check.

So how do you determine when/if the entity lied?

Did they lie the first time when they said yes, or the second time when they said no? What do you do if the entity just always says no? You still have no way of knowing if it's lying.

Story
2013-02-04, 09:58 AM
Well then you can just ask three times.


it's a pity we can't find one standard monk fix that the entire dnd community could agree on; then we could just use that. no need to post that xkcd comic, we've all seen it enough times.


It's called Tome of Battle. Also doubles as a Fighter and Paladin fix.

Pickford
2013-02-04, 10:10 AM
Well then you can just ask three times.

It's called Tome of Battle. Also doubles as a Fighter and Paladin fix.

Again: How do you 'know' which answer is truthful?

Answerer
2013-02-04, 10:12 AM
Again: How do you 'know' which answer is truthful?
Statistics can answer this question. With an intelligence score higher than any actual human being has ever had, the Wizard can figure it out.

Xaotiq1
2013-02-04, 10:14 AM
If I may be so bold...

http://www.giantitp.com/forums/showthread.php?p=14616502#post14616502

Story
2013-02-04, 10:20 AM
Again: How do you 'know' which answer is truthful?

You take the majority vote and accept the 4% chance of being wrong. If it's really important, you can ask more than that, but 96% is good enough in more cases.

Pickford
2013-02-04, 10:24 AM
Statistics can answer this question. With an intelligence score higher than any actual human being has ever had, the Wizard can figure it out.

So you want to take the risk by trying to cast it 5 times and then hope, through examination of the curve, to figure out which answer is truthful, which is a lie, if the entity doesn't know (truthfully) or if they don't know but try to lie?

This is of course assuming the entity doesn't get fed up with you and just start screen it's planar calls/come kill you. Possible, but unlikely. It doesn't matter how smart the Wizard is, you can't show statistical certainty without enough of a sample size. And given that you're contacting 'an' entity (not even necessarily the same entity each time) you'll never achieve such certainty.

Story
2013-02-04, 10:41 AM
96% is considered statistical certainty in most fields.

Big Fau
2013-02-04, 10:46 AM
People hate on monks and like wizards because on a forum you can pretend to be a schroedinger's wizard (i.e. you can always do everything) whereas in actual game play you get 4 spells per level (5 if you're a specialist, but then you can't even cast two spell-schools).

For the record, with 4 spells/spell level I can take over a continent. I've yet to see Monk 20 take more than a house. Schrödinger's Wizard isn't the Unicorn of D&D, it's an actual thing. Uncanny Forethought is the proof, and that's just if I want to prove it exists. In all actuality most Wizards follow a specific list of spells they prepare which, while not perfect, provide a wide array of answers for a positively staggering number of encounters.

Your crack about specialists shows that you don't really understand how little Evocation/Enchantment matter. Conjuration is THE best school of magic, capable of doing almost anything you need. Transmutation is a close second, to the point that those two schools alone can make you nigh unbeatable. The rest is just in case. I've played GOD Wizards, and I've DMed for them. Monks don't even compare at all.

It isn't that Schrödinger's Wizard exists, it's that it isn't necessary to be better.

Flickerdart
2013-02-04, 10:53 AM
I've yet to see Monk 20 take more than a house.
House? Is that like a dire Gazebo?

Elderand
2013-02-04, 11:08 AM
Peoples who argue that the monk is bad because of fluff are not getting my point I feel. I agree that the monk doesn't fit out of the box with the rest of the phb content. But that's not really relevant.
Dnd is a system of rules were the setting is somewhat tied to them. Somewhat, but refluffing is easy. Yes the monk doesn't fit fluff wise with the standard (and barebone) feel of everything else in the phb.
That doesn't make it bad, that makes it out of place until someone decide to play in another setting where the monk fits.

I'd argue that not everyone play in the "official" setting of the phb anyway. Making fluff entirely irrelevant. The only thing that is relevant accross all possible setting (baring house rules) is mechanics and game design.

Fluff can be a concern in other games where the rules are entirely tied down to their setting (warhammer 40K to give an exemple). There bad fluff is a very valid reason why something is bad.

Dnd is not a generic system but it can very easily be fitted to any number of settings, as exemplified by the host of published settings with very different flavors from one another.

A valid simile for the monk is this. You have several cans of paints, they all are shiny pastel except one who is muddy brown. Everyone argue the muddy brown one is bad paint, but they don't do it because it's muddy brown, they do it because it's lead based and laced with asbestos.

There is no point to discussing fluff because that's entirely dependent on every player and every dm preferences. Remove all fluff and the monk is still crap compared to most everything else. Remove all fluff and a swordsage is a good class.

Starbuck_II
2013-02-04, 11:18 AM
Really, there should have been options/paths for the Monk:
Skirmisher monks (there are people that think this is what a mank is for, Kord help them), Tank Monk (stands and fights), Mage hunter (some people like this idea), and Shoalin monk.

Give each path abilities to help them perform their duties.

Sadly, they instead decided to cram everything in one when they made 3.0/3.5 Monk resulting in nothing good.

Juntao112
2013-02-04, 11:47 AM
This is of course assuming the entity doesn't get fed up with you and just start screen it's planar calls/come kill you. Possible, but unlikely.
Also, not anywhere in the rules.


It doesn't matter how smart the Wizard is, you can't show statistical certainty without enough of a sample size. And given that you're contacting 'an' entity (not even necessarily the same entity each time) you'll never achieve such certainty.
Given that each deity of the same category has the same chance of telling you misinformation, I think you could wrangle it.


Is it me or do Dwarves make great monks? Bonus to Con, resist tripping and bull rush, saves, and so forth. Not only can you do the Fistbeard Beardfist thing for x2 Con mod to AC, but if you choose an Azerblood (descended from a fire elemental and a dwarf), you could take levels in Tattooed Monk and use Alter Self to turn into a Dwarven Ancestor (http://www.wizards.com/default.asp?x=dnd/ex/20060704a&page=3). Large size and +18 NA goes a long way towards solving many of the Monk's problems, especially when combined with Invisible Fist.

Snowbluff
2013-02-04, 01:01 PM
96% is considered statistical certainty in most fields.Dammit, it'll a cold day in Warlock-land that I learn anything here! *Covers ears*


House? Is that like a dire Gazebo?

It's actually a term for a low-HD Mansion.

Story
2013-02-04, 01:01 PM
Or you could be a level 3 Wizard and turn into a Dwarven Ancestor.

Pickford
2013-02-04, 01:14 PM
For the record, with 4 spells/spell level I can take over a continent. I've yet to see Monk 20 take more than a house. Schrödinger's Wizard isn't the Unicorn of D&D, it's an actual thing. Uncanny Forethought is the proof, and that's just if I want to prove it exists. In all actuality most Wizards follow a specific list of spells they prepare which, while not perfect, provide a wide array of answers for a positively staggering number of encounters.

Your crack about specialists shows that you don't really understand how little Evocation/Enchantment matter. Conjuration is THE best school of magic, capable of doing almost anything you need. Transmutation is a close second, to the point that those two schools alone can make you nigh unbeatable. The rest is just in case. I've played GOD Wizards, and I've DMed for them. Monks don't even compare at all.

It isn't that Schrödinger's Wizard exists, it's that it isn't necessary to be better.

The downsides being: It costs a feat, you have to have the slot open ahead of time (i.e. empty), it takes a full-round action to cast (no movement, can't use meta-magic) and it casts at -2 CL.

As for the 'god wizards'...I guess I'm just not particularly fazed, reaching that point requires several assumptions, including ignoring statistical certainties. Assuming there's at least one diviner in the entire multi-verse capable of casting Foresight, anyone trying to become a pun-pun would be insta-gibbed the second they used the candle of invocation. (This would trigger the warning effect of Foresight giving the Diviner time to Greater Teleport and cast Imprisonment = No pun-pun...ever. Ever.)

Edit:

Also, not anywhere in the rules.

OH yeah, no where in the rules....well...except in the description of the spell:

"they resent such contact" and "this divination may be blocked by an act of certain deities or forces."

Hence: Screening their calls. And if one can argue that an efreeti might try to take revenge for being briefly planar bound it is just as likely if you try to play 20 questions with a 'god' that they will snuff out your mortal just to prevent you annoying them anymore.

JaronK
2013-02-04, 01:25 PM
Luckily there are ways to block most divinations. Also, Foresight only applies moments in advance. If someone wants to start the Pun Pun loop, they'd be smart enough to gain all the power, then hold on to it without doing anything to anyone for a long time (long enough to clear the future sight abilities of most gods) just to lie low.

And of course one of the first abilities they can give themselves is complete immunity to even deity level detection.

JaronK

TopCheese
2013-02-04, 01:26 PM
(Its a punchbarian.)

Oh no!
Oh no!

OH YEAAAAAAH! *RAGES* *BURST THROUGH WALL*

(Kool Aid is a type of punch right?)

Pickford
2013-02-04, 01:31 PM
Luckily there are ways to block most divinations. Also, Foresight only applies moments in advance. If someone wants to start the Pun Pun loop, they'd be smart enough to gain all the power, then hold on to it without doing anything to anyone for a long time (long enough to clear the future sight abilities of most gods) just to lie low.

And of course one of the first abilities they can give themselves is complete immunity to even deity level detection.

JaronK

Foresight actually doesn't list how far in advance it provides the warning, just 'impending'. But...It does say that it grants a warning far enough in advance to allow the character to act.

So with that being the only caveat, if the character 'needed' to act years in advance to avoid their untimely demise, it would warn them to this effect.

Meaning if you even 'thought' about pulling a pun-pun, you'd be dead basically immediately.

In essence, pun-pun is pun-possible. doh. impossible.

Story
2013-02-04, 02:22 PM
As for the 'god wizards'...I guess I'm just not particularly fazed, reaching that point requires several assumptions, including ignoring statistical certainties. Assuming there's at least one diviner in the entire multi-verse capable of casting Foresight, anyone trying to become a pun-pun would be insta-gibbed the second they used the candle of invocation. (This would trigger the warning effect of Foresight giving the Diviner time to Greater Teleport and cast Imprisonment = No

Stop going after strawmen. Pun-pun is a TO excercise. 'God' Wizard refers to the strategy of using your powers for buffing and BFC so that you don't obviously outshine your lower tiered companions.

Anyway, it's rather tiring debating someone who refuses to do any research. Just look up an arena battle or something if you think high level monks are good. Or just read the Tier List and its explanations.

Karnith
2013-02-04, 03:10 PM
Specifically, the use of "god wizard" refers to playing a wizard in line with recommendations from this handbook (http://brilliantgameologists.com/boards/index.php?topic=394.msg8043#msg8043), though it and Logicninja's guide to being Batman (http://www.giantitp.com/forums/showthread.php?t=104002) are frequently conflated, since both involve playing hyper-competent wizards.

When people on this board (and other optimization boards) refer to God wizards, they are not talking about Pun-Pun or other TO cheese, but wizards who use their spells intelligently and effectively to solve encounters. This places them in contrast to hyper-specialized wizard builds, like the Mailman, that are effective but not very flexible, and also to wizards who are played well below the class's potential ("I'll prepare fireball in all of my 3rd-level spell slots!"). God wizards are considered extremely powerful because of the sheer variety of options that they have access to, not because of absurd TO shenanigans.

Juntao112
2013-02-04, 03:12 PM
OH yeah, no where in the rules....well...except in the description of the spell:

"they resent such contact" and "this divination may be blocked by an act of certain deities or forces."

Hence: Screening their calls. And if one can argue that an efreeti might try to take revenge for being briefly planar bound it is just as likely if you try to play 20 questions with a 'god' that they will snuff out your mortal just to prevent you annoying them anymore.

Nowhere in the rules was directed at the part where you said "come kill you", as you yourself have just proven. I apologize for the confusion.


Or you could be a level 3 Wizard and turn into a Dwarven Ancestor.

But then you're a) not a monk and b) not beating people up with your fists of stone.

Flickerdart
2013-02-04, 03:14 PM
God wizards are considered extremely powerful because of the sheer variety of options that they have access to, not because of absurd TO shenanigans.
Of course, it's very common for people new to online D&D discussions to have a much lower bar of what counts as absurd TO shenanigans, simply because their playgroup has never thought of using those spells in that way. "Monk beats wizard" is a lot easier to believe when the wizards one is used to never cast any spells with a duration longer than rounds/level.


But then you're a) not a monk and b) not beating people up with your fists of stone.
Fist of Stone is a 1st level spell.

Randomguy
2013-02-04, 03:43 PM
Most actually would, as mentioned.



The first two levels of Monk are the best; after that it becomes increasingly difficult to justify it. Saying "oh Monk is fine, you can put two levels on a monster and it'll be great!" is kind of seriously missing the point that a base class has 20 levels.

My point wasn't that "monks are fine". My point is that "A couple of monk levels make a good power boost for some monsters". I agree that monks are pretty seriously underpowered.

HMS Invincible
2013-02-04, 03:53 PM
Oh Pickford, I can't tell if you're trolling or if this was how you played in your game. Of course your DM will try make you cool, it'd be a crappy game if you showed up every week to be schooled by your DM.

Tell me how you would possibly deal with these scenarios (TM):
Situation 1: A Black Dragon has been plaguing an area, and he lives in a trap filled cave. Deal with him.

Situation 2: You have been tasked by a nearby country with making contact with the leader of the underground slave resistance of an evil tyranical city state, and get him to trust you.

Situation 3: A huge army of Orcs is approaching the city, and should be here in a week or so. Help the city prepare for war.

You may get help from your hypothetical party members, but note what your contribution is. aka, If your contribution is I inform the wizard and he summoned his army of solars, that isn't selling your class.
Now tell me how an equal level commoner with your point buy, and WBL does it differently. Note, the monk should* be better off, but are you better enough to make it worth being alone in a monastery while everyone else was out having fun with the girls?
Nobody else spoil the answers, googling is cheating. =P

Edit: The level of the scenario is your choice, but impressing others is a plus.

Juntao112
2013-02-04, 03:57 PM
You should probably indicate the level at which this all takes place.

Story
2013-02-04, 03:57 PM
Well the Monk is more likely to survive the traps at least.

As for level, does level 10 sound fair? That's about the point where the position of the low tier characters starts to become untenable without serious optimization.

Juntao112
2013-02-04, 03:58 PM
Please. Chicken Infested Commoner can spring the traps from a distance.

He can also help feed the city's defense forces during the invasion.

Morcleon
2013-02-04, 03:59 PM
Well the Monk is more likely to survive the traps at least.

Why would the wizard even need to risk himself triggering the traps? Just send in a summoned creature... :smallbiggrin:

Story
2013-02-04, 04:00 PM
Oh right, forgot about that.

Of course, I think summoning more than say, 3 chickens per round should be considered TO.


Why would the wizard even need to risk himself triggering the traps? Just send in a summoned creature... :smallbiggrin:

I was comparing the Monk to the Commoner. Of course the Wizard breezes though it.

GreenSerpent
2013-02-04, 04:04 PM
It says a lot when the best Monk builds have as few levels of Monk in them as possible.

Juntao112
2013-02-04, 04:04 PM
Come to think of it, the commoner could get himself sold into slavery and make contact with the leader of the resistance that way. He could obtain trust via chickens, because who wouldn't trust a man who can produce chickens at will?

Morcleon
2013-02-04, 04:06 PM
I was comparing the Monk to the Commoner. Of course the Wizard breezes though it.

Oh. Then buy a mule. Or several. Push them through the traps. :smallamused:

Story
2013-02-04, 04:11 PM
Handle Animal as a class skill comes in handy.

GreenSerpent
2013-02-04, 04:15 PM
http://www.giantitp.com/forums/showthread.php?t=232822

The Commoner Handbook. Might provide a little insight into how commoners can actually be quite dangerous at level 1.

Lans
2013-02-04, 04:37 PM
It's called Tome of Battle. Also doubles as a Fighter and Paladin fix.

It also fixes barbarians, but they were more of a car that was missing a headlight so it wasn't that big of deal.

holywhippet
2013-02-04, 04:46 PM
Though, in 3.5, Psionics can definitely improve the Monk. Tashalatora on an Ardent or Psychic Warrior does very well.

Yeah, but that leads me to the obvious question of exactly what class you are playing if you build up a monk like that. There are a lot of classes which can be combined with monk to make them into an unarmed killing machine. But you aren't really playing a monk any more when you do that. You could just as well play a fighter or barbarian who only fights unarmed and unarmoured and call that a monk.

If I was going to go the "monk" route in a 3.5 game again I'd likely take two levels of monk then switch to either cleric or druid and go the sacred fist route. But again, I'd be adding the power of divine spells to the build which is like attaching explosives to an arrow. It's turning something kind of strong into something crazy strong using methods far different to the original concept.

Oscredwin
2013-02-04, 04:53 PM
If I was going to go the "monk" route in a 3.5 game again I'd likely take two levels of monk then switch to either cleric or druid and go the sacred fist route. But again, I'd be adding the power of divine spells to the build which is like attaching explosives to an arrow. It's turning something kind of strong into something crazy strong using methods far different to the original concept."Far different to the original concept," but strong enough to team up with Batman (https://www.youtube.com/watch?v=TkkyualjFHA).

Answerer
2013-02-04, 04:57 PM
The thing is, the Monk class is incredibly vaguely defined, conceptually (this is actually the source of its problems). Tashalatora'd Psychic Warrior can't objectively be said to be "less monkly" than the Monk because what exactly it means to be a monk is largely undefined. It depends on how you see the class.

Put another way, the Monk class doesn't do much. As soon as you want to improve it, you have to give it things to do. But because the original didn't really do anything, you have no real guidance to what things should be added.

In the end, I strongly feel it doesn't matter, though. The class is "monkly" if it accomplishes the things that the player wanted to accomplish when he thought "I want to be a monk." If that means you want to be very explicitly religious, the Cleric/Sacred Fist route makes sense. If you want to be the mystic martial artist, then either Swordsage or Psychic Warrior might make more sense. If Monk was just supposed to be an unarmed warrior, then maybe Fighter/(refluffed) Barbarian (Rage as Zen Focus or something) makes more sense.

Big Fau
2013-02-04, 04:58 PM
The downsides being: It costs a feat, you have to have the slot open ahead of time (i.e. empty), it takes a full-round action to cast (no movement, can't use meta-magic) and it casts at -2 CL.

The feat cost is minor (as Wizard bonus feats can be spent on Spell Mastery), the open slot is a non-issue (if you have the feat, odds are you are going to leave a few slots open so you can use the feat), the full-round action is not nearly as bad as a full round, and the -2 CL is so trivial it might as well not exist.


As for the 'god wizards'...I guess I'm just not particularly fazed, reaching that point requires several assumptions, including ignoring statistical certainties. Assuming there's at least one diviner in the entire multi-verse capable of casting Foresight, anyone trying to become a pun-pun would be insta-gibbed the second they used the candle of invocation. (This would trigger the warning effect of Foresight giving the Diviner time to Greater Teleport and cast Imprisonment = No pun-pun...ever. Ever.)

Where the Nine Hells did you get Pun Pun from GOD Wizard? One is TO, the other is a completely unrelated spellcaster. (http://community.wizards.com/go/thread/view/75882/19873034/Treantmonks_guide_to_Wizards:_Being_a_God&post_num=1#338493474)

Seriously, at what point in my previous post did you manage to assume I was even referencing Pun Pun?

lsfreak
2013-02-04, 05:08 PM
Batman Wizard: A wizard that uses their two tools at their disposal (spells and allies) to end encounters quickly and easily. Prone to spotlight-stealing, jerkassery.

GOD Wizard: A wizard that uses their spells to support their allies and end encounters quickly and easily. Deliberately not spotlight-stealing.

Pun-Pun: A convoluted way that abuses the rules to literally get unlimited power.

Juntao112
2013-02-04, 05:12 PM
Batman Wizard: A wizard that uses their two tools at their disposal (spells and allies) to end encounters quickly and easily. Prone to spotlight-stealing, jerkassery.

You have no idea. (http://superdickery.com/index.php?option=com_content&view=article&id=1314:batman-kills-robin-again&catid=30:frames-and-panels-index&Itemid=34)

Answerer
2013-02-04, 05:20 PM
Batman Wizard: A wizard that uses their two tools at their disposal (spells and allies) to end encounters quickly and easily. Prone to spotlight-stealing, jerkassery.
You have clearly never actually read TLN's guide. He specifically recommends against spotlight-stealing. His argument against Evocation includes the idea that you're stepping on others' toes. He tells people not to use Polymorph effects. He points out prestige classes that are overpowered. And so on.

People's idea of what the Batman Wizard is, and what TLN actually wrote, seem to be very different things.

Psyren
2013-02-04, 05:33 PM
"A good wizard primary caster is like a good bassist - you only notice him when he leaves."

This is true in both TLN and TM's guides.

lsfreak
2013-02-04, 05:38 PM
You have clearly never actually read TLN's guide. He specifically recommends against spotlight-stealing. His argument against Evocation includes the idea that you're stepping on others' toes. He tells people not to use Polymorph effects. He points out prestige classes that are overpowered. And so on.

People's idea of what the Batman Wizard is, and what TLN actually wrote, seem to be very different things.


To this end, you will cast spells that help you and your poor, ignorant, inferior companions (read: party), and hamper your enemies.

Leave damage to the guys with pointy sticks; you have better things to do.

-Augment Summoning: if you're summoning regularly, you're doing something wrong. That's the druid's or cleric's job; after all, every time a wizard casts a spell that's on a divine list, for that round he's a sucker. Don't take this.

--Direct Damage: These spells, by and large, suck. Occasionally, they're useful, but when a good mage wants something damaged, he tells the fighter to go hit it. If it's hard to hurt, he buffs the fighter first.
Whether or not it's just part of his humor, it can come across as "be ruthlessly efficient," not "don't spotlight steal."

RFLS
2013-02-04, 05:38 PM
Yeah, but that leads me to the obvious question of exactly what class you are playing if you build up a monk like that. There are a lot of classes which can be combined with monk to make them into an unarmed killing machine. But you aren't really playing a monk any more when you do that. You could just as well play a fighter or barbarian who only fights unarmed and unarmoured and call that a monk.

You mean....we're playing a game based around imagination...and the fluff is mutable? Hot diggity.

Oscredwin
2013-02-04, 05:47 PM
Whether or not it's just part of his humor, it can come across as "be ruthlessly efficient," not "don't spotlight steal."

See, I always read that as "feel awesome and don't step on anyone's toes". A fighter guide written in the same style would say things like "let yourself get buffed before charging in, it makes the wizard feel like he's doing things," and "Martial Study: Mountain Hammer, if you're breaking down doors, you're doing it wrong. Let the rogue be your personal doorman, use that strike to crush the monster with crazy DR that's pounding on the rogue".

Starbuck_II
2013-02-04, 06:05 PM
See, I always read that as "feel awesome and don't step on anyone's toes". A fighter guide written in the same style would say things like "let yourself get buffed before charging in, it makes the wizard feel like he's doing things," and "Martial Study: Mountain Hammer, if you're breaking down doors, you're doing it wrong. Let the rogue be your personal doorman, use that strike to crush the monster with crazy DR that's pounding on the rogue".

Someone make this guide. Make one fore the Monk as well while we are at it.

Psyren
2013-02-04, 06:21 PM
Whether or not it's just part of his humor, it can come across as "be ruthlessly efficient," not "don't spotlight steal."

Since when are those mutually exclusive? :smallconfused:

In fact, one can easily argue that NOT stealing the spotlight IS being ruthlessly efficient. Think about it - if you do the fighter's job (killing things) you aren't just ruining his fun, you're wasting his actions. And burning spell slots to do it. Ditto on unlocking doors/chests when you have a rogue.

holywhippet
2013-02-04, 06:59 PM
The thing is, the Monk class is incredibly vaguely defined, conceptually (this is actually the source of its problems). Tashalatora'd Psychic Warrior can't objectively be said to be "less monkly" than the Monk because what exactly it means to be a monk is largely undefined. It depends on how you see the class.


My mental image of the monk pretty much starts with Bruce Lee. Basically you want someone who is extremely good at hand to hand combat - especially unarmed. Most opponents can't hit them because before they have even finished swinging at them they've been punched twice and a third punch is en route.

Then add some Cain from the TV show Kung Fu - quite well educated in his own area, wise and with all sorts of fancy mystical tricks he can pull off due to his training.

The monk as given covers parts of these but not all of these. You have flurry of blows for "faster" blows, but with 3/4 BAB you aren't going to hit as often. Did Bruce Lee miss many attacks? No, he didn't. The lower AC from not wearing armour means you get hit more often - again, Bruce Lee didn't take many beatings.

One thing I've seen in the Final Fantasy Tactics video game which I think would be appropriate to a monk is the hamedo ability. In that game it let you hit an opponent who is about to attack you before they can get their attack in.

Phelix-Mu
2013-02-04, 07:01 PM
Wow, so this got a little off track, I've noticed. Lemme contribute a little more two-cents to the GDP of Columbia we are collecting here:

Why are we comparing monk to wizard? Talk about a comparison likely to end unfavorably for the monk. The only three interactions I think we need to consider between the two classes are:

1.) Multiclass: Monk and wiz are much cooler if combined. Greater mighty wallop is cool! Cast it on yourself for bonus points! Hurray!

2.) Make friends: Monk should have a wizard party-member, ally, or just contact. Wizards are cool. Any cool monk should know a wizard and be on good terms. Seriously extends the survivability of any class to have wizard friends. And wizzies can often AoE over the monk without worrying too much (as long as it isn't the super nuke AoEs or no-saves)...good crowd control option with monk on mop-up.

3.) Nemesis: Oooh, so the enemy is a powerful, high-tier caster! This is bad for the lone monk, so be sure to travel with a party of classes that are played in a cooperative manner. Anyone that considers class v class to be anything more than an amusing and somewhat instructive thought experiment is disregarding much of the way in which the game was meant to be played (i.e., with friends and for fun).

Pickford fights a noble but fruitless fight. The rules support for magic that came out consistently in almost every book mean that, if even a small portion of them are loophole-rific and exploitable, then full casters can dominate in a way that is terrifying from a game-balance perspective, all op being equal. Magic is supposed to be an important component of the D&D-themed world, but eventually they expanded what was possible to encompass doing what any other class was supposed to be good at better, faster, cheaper, more reliably than that other class. Personally, I favor druid over wizard, and druid/wiz/arcane hierophant basically is the capstone of "wait, is the ruleset dead yet? get the stick, beat it more!" The limits of high-level magic in the hands of high-tier casters are essentially limitless, as every day they get a set of powers that extend far beyond what laborers, armies, or mighty rulers could accomplish by other means. The only hope is carefully-planned ambush and some kind of weakness or blindspot on the part of caster (luckily most actual npcs, and even characters, have something of this kind).

Anyway, I think monk deserves a good fix, has little of this in any official treatment, and I still love to play one. I like challenges, what can I say? You may not like the flavor, you may not like the mechanics (rightfully so), but there are a number of other rather terrible classes and we should pick on them more and monk less!

Or we can just keep talking about wizard....

Gigas Breaker
2013-02-04, 07:05 PM
When I think of a martial artist I think of kung fu movies with multiple fighting styles. I feel like tashalatora and swordsage cover that aspect pretty well.

GreenSerpent
2013-02-04, 07:06 PM
As a small point, I'm currently working on a Monk rewrite based around the Monk having complete control of their own body. Hair to toes. The problem is figuring out how to boost it up in power but still try and keep the feel of the class.

The problem as many people have pointed out is it's designed for both flurrying and moving about, and working those together is tricky.

holywhippet
2013-02-04, 07:21 PM
1.) Multiclass: Monk and wiz are much cooler if combined. Greater mighty wallop is cool! Cast it on yourself for bonus points! Hurray!


In terms of game mechanics it is a good idea. There are a few arcane spells that can really help out a monk - mage armor especially. Thing is, it means the monk only becauses good if they have access to arcane magic either by casting it themself or getting it from somewhere else. Each class should be able to be self sufficient to be effective with cross class synergy to be the icing on the cake.

Karnith
2013-02-04, 07:23 PM
Why are we comparing monk to wizard? Talk about a comparison likely to end unfavorably for the monk. The only three interactions I think we need to consider between the two classes are:
1.) Multiclass: [...]
2.) Make friends: [...]
3.) Nemesis: [...]
Well, those, and probably the most important comparison for players, which is whether or not a monk is appropriate to play in a game. Monks can be fine in low-powered games, but they are horrifically underpowered compared to most other classes, and can often find themselves unable to contribute meaningfully without unreasonable amounts of help from the party or DM intervention, as the perpetual monkday threads attest to.

The rules support for magic that came out consistently in almost every book mean that, if even a small portion of them are loophole-rific and exploitable, then full casters can dominate in a way that is terrifying from a game-balance perspective, all op being equal.
Actually, even if none of their options are broken, wizards (and clerics, druids, etc.) would still be extremely powerful because they have so many options, and other classes have so few in comparison. Even banning stuff like Gate, Shapechange, Celerity, etc. leaves them with the ability to target all three of an enemy's saves with save-or-dies, lots of ways to control the battlefield, tons of utility spells that they can change from day to day, ways to buff the party and protect themselves, summon/create minions, deal with social situations, and so on and so forth.

By contrast, our friend the monk isn't particularly good at any role without heavy optimization (or multiclassing, I guess), and has very little versatility in how he solves problems ("Can I solve this problem by punching something? How about by speaking a different language? No? Well, I guess I'll go back to carrying everybody's stuff").

The problem as many people have pointed out is it's designed for both flurrying and moving about, and working those together is tricky.
Presumably pouncing flurries would help make the class abilities feel less disjointed?

Phelix-Mu
2013-02-04, 07:33 PM
As a small point, I'm currently working on a Monk rewrite based around the Monk having complete control of their own body. Hair to toes. The problem is figuring out how to boost it up in power but still try and keep the feel of the class.

The problem as many people have pointed out is it's designed for both flurrying and moving about, and working those together is tricky.

I mentioned before that a suggestion for monk fixes thread I had had a suggestion to allow monks to move 1/2 their movement as a swift action. This is a fix that allows jumping around and such while not augmenting the functionality of flurry. Other ideas like standard action flurry and AoO flurry might be very complicated.

Institute the full BAB and wisdom to attack/damage and leave flurry alone is the simple approach. The ability is complicated enough without additional tinkering.

But I do love me some tinkering....

To someone else' comment about the multiclass thing, monks can be self-sufficient, but rely a lot more than other classes on combat avoidance, a strategy that, in most d&d campaigns, isn't going to be that popular or effective (since plot challenges often can only be avoided to a certain degree). I totally agree that adding the wizard dip doesn't speak highly of straight monk, specifically for a class with alignment and multiclass restrictions that are kind of geared to make single-classing more attractive.

In my first post in this thread I referenced an earlier thread of mine that addressed what people thought would be simple ways to improve monk. Got some good suggestions.

Kazyan
2013-02-04, 07:38 PM
Someone make this guide. Make one fore the Monk as well while we are at it.

I think the snark would be more convincing with a bit of variety, so let's pick a class that has some more. Alfred: The Sophisticated Guide to Barbarians.

"Eagle Spirit Totem: You could. Your poor buffbot caster buddy is jumping through hoops to get Mindsight, though. If you're concerned about golems hitting him, fine, because what could your buddy possibly do? Orbs, grease, and summons don't count for the same reasons that crossbows, marbles, and hirelings don't."

"Improved Uncanny Dodge: You've been trading out all of your trap-related stuff to give the rogue some breathing room, remember? Trade this for a bonus feat (Sandstorm) that makes your damage even more gentlemanly."

Clericzilla
2013-02-04, 08:04 PM
My mental image of the monk pretty much starts with Bruce Lee. Basically you want someone who is extremely good at hand to hand combat - especially unarmed. Most opponents can't hit them because before they have even finished swinging at them they've been punched twice and a third punch is en route.

Then add some Cain from the TV show Kung Fu - quite well educated in his own area, wise and with all sorts of fancy mystical tricks he can pull off due to his training.

The monk as given covers parts of these but not all of these. You have flurry of blows for "faster" blows, but with 3/4 BAB you aren't going to hit as often. Did Bruce Lee miss many attacks? No, he didn't. The lower AC from not wearing armour means you get hit more often - again, Bruce Lee didn't take many beatings.

One thing I've seen in the Final Fantasy Tactics video game which I think would be appropriate to a monk is the hamedo ability. In that game it let you hit an opponent who is about to attack you before they can get their attack in.


I agree though I would combine Bruce Lee, Chuck Norris (enter the dragon was a fluke.. only because he was curious as to what they felt like), and Jackie Chan.

Make Flurry of Blows a touch attack and we might be getting close to getting somewhere.

Any chance we can get this Monk thread stickied? This questioned is asked a ton of times... -_-;;;

zlefin
2013-02-04, 08:04 PM
so, i wonder how many different conceptions there are of what a monk is supposed to be.
As well as what sort of role a monk is supposed to play in a party.

Answerer
2013-02-04, 08:30 PM
My mental image of the monk
Stop, stop right there. That right there is where the problem with your previous post is: it's based on your conception, and then says that certain options (Psychic Warrior for instance) are "less monkly" because they don't match this. But this, your mental image, is not even remotely universal.

I'd see a modern, real life martial artist, particularly a movie-star one (not as a debasement of their skills, but on their outlook on life: they are not, ya know, meditating in a monastery, they're out being awesome and famous and making lots of money) as being a Fighter with the appropriate feats than a Monk.

Snowbluff
2013-02-04, 08:33 PM
Any chance we can get this Monk thread stickied? This questioned is asked a ton of times... -_-;;;

Not feeling your changes bro. This thread doesn't have enough anti-monk tendencies to make the point clear.

Lans
2013-02-04, 09:06 PM
so, i wonder how many different conceptions there are of what a monk is supposed to be.

This actually leads to one of my preferred monk fixes, take any monk prestige class and give its abilities to the monk, starting at level 1. When that prestige class gets capped out extrapolate it, or add another, or both! Change the DCs to 1/2 monk level

GenericMook
2013-02-04, 09:23 PM
This actually leads to one of my preferred monk fixes, take any monk prestige class and give its abilities to the monk, starting at level 1. When that prestige class gets capped out extrapolate it, or add another, or both! Change the DCs to 1/2 monk level

Actually, that's kinda clever. Makes the monk less of a guy that knows some techniques that every other monk knows, and more of a master of one or more styles.

toapat
2013-02-04, 09:26 PM
Someone make this guide. Make one fore the Monk as well while we are at it.

wouldnt that guide consist mostly of advice that involves finding the nearest Meatgrinder/Woodchipper/Frenzied Berzerker/Lava Lake and jumping into it untill reroll, then make an Unarmed Swordsage?

Karnith
2013-02-04, 09:32 PM
wouldnt that guide consist mostly of advice that involves finding the nearest Meatgrinder/Woodchipper/Frenzied Berzerker/Lava Lake and jumping into it untill reroll, then make an Unarmed Swordsage?
Hey now, be fair. It would also contain advice on rerolling a Tashalatora Monk, and maybe a punchbarian.

akahdrin
2013-02-04, 09:34 PM
{Scrubbed}

MagnusExultatio
2013-02-04, 09:36 PM
{Scrubbed}

You didn't. Not even remotely.

Gigas Breaker
2013-02-04, 09:43 PM
{Scrubbed}

Except monk isn't a fun class. ;) ;) ;);)
;)))))))))))

Snowbluff
2013-02-04, 09:45 PM
[COLOR="Red"]{Scrubbed}

There is not enough blue text in the world! I fixed that for you and there was not enough, so now this is all in blue!:smalleek:

ArcturusV
2013-02-04, 09:57 PM
You know, to represent that "Bruce Lee" aspect, I almost wish 3.5 Monks had the 2nd Edition Barbarian Fighter "counterattack" ability. Percentile chance that you can cause a melee attack to miss, and get a free attack on your attacker. Least then it'd have some comparison to it's buddies Fighter and Barbarian. Fighter ignoring attacks through good AC (until AC becomes trivial), Barbarian ignoring attacks through Raging HP and DR, and Monk ignoring attacks through nimble dodges and counter attacks.

Lord_Gareth
2013-02-04, 10:00 PM
Monks are great for those that want to play a fun class.

My fun involves being able to participate at the game table in a fashion that makes me feel like the hero or villain I'm attempting to portray. Monks cannot do that. Ergo monks are not fun.

Snowbluff
2013-02-04, 10:05 PM
My fun involves being able to participate at the game table in a fashion that makes me feel like the hero or villain I'm attempting to portray. Monks cannot do that. Ergo monks are not fun.

That's an opinion! You can't have an opinion that you have come to in a rational way!:smallbiggrin:

Lans
2013-02-04, 10:10 PM
Actually, that's kinda clever. Makes the monk less of a guy that knows some techniques that every other monk knows, and more of a master of one or more styles.

It kind of speak volumes about monks effectiveness that what is essentially a limited free gestalt or broken ACF use, is necessary for them to not completely suck.

Which is only broken if used with a monk prestige class that advanced spells or psionics. If it advanced maneuvers its probably just where it should be

Psyren
2013-02-04, 10:47 PM
I can't speak to the 3.5 monk, but there are plenty of fun monks in Pathfinder. (The base one not so much, but hey.)

Lord_Gareth
2013-02-04, 10:48 PM
I can't speak to the 3.5 monk, but there are two fun monks in Pathfinder. (The base one not so much, but hey.)

Fiiiiixed that for you. Zen Archer, yes. That one with a Q, certainly. The others? No.

Snowbluff
2013-02-04, 11:18 PM
Fiiiiixed that for you. Zen Archer, yes. That one with a Q, certainly. The others? No.

Yeah, but Quigonginidc feels gimpy still and Zen Archer is like a soulbow but you can't cosplay as Zeratul for Halloween.

RFLS
2013-02-04, 11:29 PM
Fiiiiixed that for you. Zen Archer, yes. That one with a Q, certainly. The others? No.

Eh, Master of Many Styles can be entertaining, especially if you focus on an AoO build with it.

theMycon
2013-02-05, 12:04 AM
Monks are considered so bad because monks are pretty bad, and the internet is an echo chamber where people people try to sound smart by repeating popular opinions louder and harder than they heard them.

Monks are pretty bad for several reasons:
1) What they're good at (being very mobile, being relatively hard to hurt, ignoring "The DM put work into this campaign"-only challenges like poison, disease, and funny languages) are not very good. This is also why the Pathfinder monk isn't much better; it makes the monk better at what it's already good at.

2) What they're good at (being very mobile, standing still and hurting things, being relatively hard to hurt) doesn't work well with each other, and they don't help the party if the DM doesn't specifically focus on you.

3) Everything they have can be done better by magic by mid-levels. This is true of every non-caster class, mind you, but if the average party has 2 characters who can entirely replace their specialization with a standard action without specifically building for it; there isn't really a place for them.

4) Rules as intended are unclear, and Rules as written aren't much better. There's no guarantee what the DM will follow, and they can seriously screw you over or give you the sky. Is a monk proficient with their own attack? Is an unarmed attack the same thing as an unarmed strike? Do they qualify for Improved Natural Attack? What does that "no such thing as an offhand attack" line mean?

5) The PHB recommends giving a monk more Str than necessary to carry a sling without encumbrance issues. This produces a lot of poorly built monks.

Pickford
2013-02-05, 12:26 AM
Oh Pickford, I can't tell if you're trolling or if this was how you played in your game. Of course your DM will try make you cool, it'd be a crappy game if you showed up every week to be schooled by your DM.

Tell me how you would possibly deal with these scenarios (TM):
Situation 1: A Black Dragon has been plaguing an area, and he lives in a trap filled cave. Deal with him.

Situation 2: You have been tasked by a nearby country with making contact with the leader of the underground slave resistance of an evil tyranical city state, and get him to trust you.

Situation 3: A huge army of Orcs is approaching the city, and should be here in a week or so. Help the city prepare for war.

You may get help from your hypothetical party members, but note what your contribution is. aka, If your contribution is I inform the wizard and he summoned his army of solars, that isn't selling your class.
Now tell me how an equal level commoner with your point buy, and WBL does it differently. Note, the monk should* be better off, but are you better enough to make it worth being alone in a monastery while everyone else was out having fun with the girls?
Nobody else spoil the answers, googling is cheating. =P

Edit: The level of the scenario is your choice, but impressing others is a plus.

No I'm not trolling, I'm saying that each class is strengths and weaknesses. A wizard would be more at risk of having possessions stolen from their room at night or the like. (Whereas the monk would still function at virtually peak efficiency).

At any rate:
1) Frankly it depends on the level of the dragon. We fight and kill it or could collapse the cave entrance and trap it underground. I'm not sure how class skills are particularly relevant in this scenario. (How were you expecting me to answer this differently?) As a side note, a wizard could only cast summon monster V, which isn't that great. Anyone sending in summoned monsters at 10th level might not even have the monsters 'reach' the dragon (depending on cave depth, at 10th level we're talking only 60 seconds of creature time.) but if they did, all you would have achieved is possibly luring the dragon out of the cave...unless he just stayed home that day.

But...if I had to pick a monk class feature to shine here (at 10th level): I'd go with their increased movement speed. This would still be a team effort, but I'd be the bait. While my teammates setup some traps for the dragon in and around the entrance, I would go inside, alone, to throw a shruiken at it. With my +30' speed I can maintain full distance from the dragon and with improved evasion I would, at worst, only take 1/2 damage from it's breath weapon (but more likely no damage at all). I run out of the cave, tumble through the traps and, after the dragon trips them, participate in a good old fashioned beat down.

2) Sorry why does a Monk care? This is almost polar opposite against the fluff of the monk class. Monk's are in this for themselves, not a bunch of commoners who couldn't protect themselves.

A more interesting adventure (in the same scenario-verse) would be to assassinate the head of the underground resistance on behalf of the tyrannical state's rulers. This would involve gaining the trust of the leader by pretending to fight slavers (dealing non-lethal damage only) and "helping" some "slaves" (read: party members) escape. Then, having gained the trust of the foolish rebels a monk could easily gain audience "unarmed" with the leader for the actual hit.

Actually, this could work even better for a 15th level monk. Challenge the leader to a friendly boxing match. During the match the monk uses quivering palm and, hours later while the leader is meeting with someone else, wills the victim to die. The perfect kill, no one even remotely suspects and blame may come to rest on those who were alone with the rebel leader.

3) Again, why does the monk care? I suppose we could simulate the training of peasants to defend themselves using monk weapons. But then, that's not exactly the best use of a monk class feature. Instead, I send my monk to collect plague infected corpses. Because of Purity of Body the monk is safe, the monk then uses his speed to race to each water source in the direction of the orc army and spoil them with festering corpses. The Orcs, denied a source of fresh water for miles die horribly, disease taking far more of a toll than battle ever does. (Historically accurate AND clever).

As to your query about a commoner...
1) Commoner is slow, doesn't have improved evasion and his saves are awful. He gets eaten when the dragon catches him almost immediately, or turned into goo by the acid breath, or blows all his saves and is impaled on a trap.

2) The commoner can't use quivering palm and would never come close to the melee damage assuming no quivering palm. No flurry, weaker BAB.

3) The commoner dies horribly from the plague. Good job commoner.

Edit: Actually the immunity to normal disease and the 11th level immunity to poison are potentially amazing if you use them right. (i.e. Free use of horrifically dangerous poisons from Complete Scoundrel/Complete Adventurer). Just smear them on a sai and you're good to go, 0 chance of poisoning yourself. Otherwise just use contact poisons smeared on your hands. (Black Lotus Extract deals 3d6 Con damage...good night ...anyone else?) DC 35 too.

ArcturusV
2013-02-05, 12:38 AM
Number 2 isn't really opposed to what a Monk is at all. It may be opposed to your Alignment, from the side you're talking about you're playing Lawful Neutral, or Lawful Evil. A Lawful Good Monk (which should roughly be 1/3 of them out there after all) would have no problem with a mission like that.

Your first plan is almost entirely non-monk centric. I could just have a commoner with a Longbow plink the dragon from behind the traps, and the movespeed isn't really a factor. The only reason it IS a factor is that you're using shuriken which are short ranged. It's a vague plan that any class could do, and in no way does the fact that you're a monk actually help matters in the slightest. You're not even meaningfully contributing as everything that actually DOES take down a Dragon is being done by your party, not you.

Your second plan is... vague? Not likely to work. Bluff and Diplomacy aren't really Monk Skills (Well Dip is, but unlikely to be raised particularly since Charisma is the dump stat, and you're Skill Point Starved as is) so you wouldn't have the ranks to be able to pull it off. Not to mention Charisma is about the only stat that the Monk doesn't need, so likely your dump stat. At the very least you DID tag onto a Monk feature, Quivering Palm.

... or I could just have my Commoner with ranks in Profession (Chef) and Craft (Poisons), poison his meal and still be miles away when he actually croaks.

Third depends on several factors of course. But it's at least a somewhat valid plan that uses your monk features. Of course a lower level spellcaster could just cast an inflict disease or poison on said rarefied water sources which is the only water in at least 5 days march and thus cause them to risk dying out from dehydration (And hope there isn't a first level Cleric or Druid that can create food and water out of magic for troops, like any army in a DnDverse should have as it negates SOOOOOO many logistical problems)... not to mention succumbing to disease takes days/weeks and may not effectively slow down the enemy. Plus the sheer luck of just happening to find corpses who are sufficiently diseased with something particularly nasty with no actual ability to force it.

strider24seven
2013-02-05, 12:50 AM
If your contribution is I inform the wizard and he summoned his army of solars, that isn't selling your class.


Hey, the BMX-Bandit monk can obviously contribute as much as that Angel Summoner Wizard!

Kelb_Panthera
2013-02-05, 12:56 AM
Sorry to dogpile, but that's a terrible plan for fighting a dragon. Why, in the name of all things rational, would you -ever- want to lure the dragon out of a confined space? Once that beasty is in the air those shurikens and the fighter's backup bow are the only things the two of you can hit the critter with. Your shurikens are even useless unless it comes within 50ft. Your strategy is basically, "casters, deal with this!" then hide.

Acanous
2013-02-05, 01:03 AM
Your strategy is basically, "casters, deal with this!" then hide.

This is entirely fitting with my experience of a monk in the party. He'd be playing his role perfectly.

Pickford
2013-02-05, 01:06 AM
Number 2 isn't really opposed to what a Monk is at all. It may be opposed to your Alignment, from the side you're talking about you're playing Lawful Neutral, or Lawful Evil. A Lawful Good Monk (which should roughly be 1/3 of them out there after all) would have no problem with a mission like that.

The fluff I'm referring to:

"An individual monk is unlikely to care passionately about championing commoners or amassing wealth. She cares primarily for the perfection of her art and, thereby, her personal perfection."


Your first plan is almost entirely non-monk centric. I could just have a commoner with a Longbow plink the dragon from behind the traps, and the movespeed isn't really a factor. The only reason it IS a factor is that you're using shuriken which are short ranged. It's a vague plan that any class could do, and in no way does the fact that you're a monk actually help matters in the slightest. You're not even meaningfully contributing as everything that actually DOES take down a Dragon is being done by your party, not you.

I'm positing a cave with at least one turn, otherwise the dragon just comes out and fights and it really doesn't matter 'what' your class is. And no, any other class would get out-run by the dragon's 60' land speed. Only the monk is fast enough to hit and run. Actually, given enough twists/turns the monk at 12th level could solo the dragon because of the extra movement. 1 turn full move, 1 turn attack/move. The dragon never would get an attack and always show up an action too late.

Either way, it's a straight up fight (in which case the question of how a class member would deal with it is moot) or kiting which only the monk is fast enough to do.


Your second plan is... vague? Not likely to work. Bluff and Diplomacy aren't really Monk Skills (Well Dip is, but unlikely to be raised particularly since Charisma is the dump stat, and you're Skill Point Starved as is) so you wouldn't have the ranks to be able to pull it off. Not to mention Charisma is about the only stat that the Monk doesn't need, so likely your dump stat. At the very least you DID tag onto a Monk feature, Quivering Palm.

Who says anything about bluffing? The only reason someone would have to question the scenario is if there was a reason to doubt it. Because the monk can deal non-lethal damage at his option it would be extremely convincing. So no bluff check would be reasonable, under the circumstances. All a monk would have to say is that they wish to find the resistance. Boom, true and any sense motive check auto-fails.


... or I could just have my Commoner with ranks in Profession (Chef) and Craft (Poisons), poison his meal and still be miles away when he actually croaks.

And your commoner risks poisoning himself, has no way of knowing if the meal even reaches the target. How did your commoner ingratiate himself so much as to be allowed to cook for the dear rebel leader? Did you rescue him from a hot gaspacho? :smalltongue:


Third depends on several factors of course. But it's at least a somewhat valid plan that uses your monk features. Of course a lower level spellcaster could just cast an inflict disease or poison on said rarefied water sources which is the only water in at least 5 days march and thus cause them to risk dying out from dehydration (And hope there isn't a first level Cleric or Druid that can create food and water out of magic for troops, like any army in a DnDverse should have as it negates SOOOOOO many logistical problems)... not to mention succumbing to disease takes days/weeks and may not effectively slow down the enemy. Plus the sheer luck of just happening to find corpses who are sufficiently diseased with something particularly nasty with no actual ability to force it.

Sieges can take years so time really isn't an issue on this. Once disease takes root the clerics would be overwhelmed.

You might just as well be saying: Oh the army has X to counter Y. It comes across as sour grapes.

Blinding Sickness - Ingested 1d3 days.

Then there's always the homebrew diseases that reflect common real world diseases. Many diseases have incubation periods of only a few days. That's plenty of time to incapacitate or decimate an army.

Pickford
2013-02-05, 01:08 AM
Sorry to dogpile, but that's a terrible plan for fighting a dragon. Why, in the name of all things rational, would you -ever- want to lure the dragon out of a confined space? Once that beasty is in the air those shurikens and the fighter's backup bow are the only things the two of you can hit the critter with. Your shurikens are even useless unless it comes within 50ft. Your strategy is basically, "casters, deal with this!" then hide.

Oh stop clutching your pearls. If the dragon takes to the air, go back inside the cave. If it wants to fight it has to come back inside. If not, loot it's horde and call it a day.

Carth
2013-02-05, 01:13 AM
A 12th level monk soloing a dragon of what CR?

Psyren
2013-02-05, 01:20 AM
Fiiiiixed that for you. Zen Archer, yes. That one with a Q, certainly. The others? No.

By "plenty" I'm referring to multiple combinations. So Qinggong by itself, or Qinggong/Hungry Ghost, or Qinggong/Sensei, are three different monks in my eyes.

Besides, I find Tetori to be fun too, and Vow of Silence/Truth are easy enough buffs to take.

ArcturusV
2013-02-05, 01:23 AM
Least you're throwing out reasons now why your plan might be more relevant. So there's a plus. Though the infinite Shuriken Kiting I don't see working out. Note that I'm not just pulling "Random counters out of a hat" or shaping situations based entirely on the aspect of just countering your plan. I'm doing reasonable things that any DnDverse should account for. Food and Water, for the army example, is a low level cleric spell that 1 in about 250 people can cast if I roughly remember my World Building Demographics as listed in the books. Any army capable of sacking a city would have more than enough people to be including spellcasters (After all, they are infinitely useful, even low level ones). Just like modern armies are taught to boil water and purify it before drinking, you can expect that a mythical DnDverse army would have spellcasters able to cast "Purify Food and Drink" several times a day. Which covers more than enough space at 1 cubic foot/level to cover their logistical needs (8 gallons per casting, which could work for 64 people, even a first level cleric could cast it 4 times a day for roughly 256, covering the demographic spread), and would instantly invalidate your plan. And is just basic smarts that you'd expect any army to do. Not even like it's a rare splatbook or anything, PHB. Not to mention that in a DnDverse you don't take cities by Siege. You take them by force. With the presence of Magic, a simple stone wall is NOT going to hold off an army indefinitely. And that's even IF the city has it. A lot of DnD cities don't have anything nearly as robust as stone wall 20 feet thick and 40 feet high, etc. You can't plan to hold them off for days. Hours maybe. They're going to rush you. It's a given. You're not going to have an army sitting outside your doors for a year.

But the dragon example? You don't have unlimited shuriken, and they do **** damage. A dragon gets poked for that 1d3, it's going to look at you, maybe shoot off a breath weapon. Which is going to hurt for a lot more unless you save. You run off. The Dragon, having learned you are not a threat to it in the least with your piddly 1d3 damage... sits there. To bait like that you have to prove yourself a danger, or give them a reason to chase just beyond "I exist". Dragons typically aren't stupid. They're going to figure if a lone nutter tries to attack them with a meaningless attack, and run away, it's probably a distraction so someone can steal their horde, or a trap.

So you come back in. Throw another 1d3. It breath attacks you again. Maybe casts a spell which your evasion won't apply to if you successfully saved, because, again, they are rocking enough INT to be able to realize something isn't working and adapt. In the end it's more likely you'd have to come back and run the risk of failing a saving throw and getting hurt A LOT more than 1d3 you're dealing. Even if you always save, it'll just find a way to negate your yo-yo, in and out plan. Cause a cave in and collapse the tunnel you are using to run in and out while you are IN. Now you're trapped with an angry, annoyed Dragon, alone. With you having suboptimal damage and combat options. Or do it while you are outside, and make it so you can't get back in without digging/blasting. At which point it can have something really nasty arranged for you.

strider24seven
2013-02-05, 01:27 AM
A 12th level monk soloing a dragon of what CR?

2- White Wyrmling.
Then:
???- Dragonwrought Loredrake Kobold Sorcerer with Greater Draconic Rite of Passage.

Kelb_Panthera
2013-02-05, 01:31 AM
Oh stop clutching your pearls. If the dragon takes to the air, go back inside the cave. If it wants to fight it has to come back inside. If not, loot it's horde and call it a day.

So lure him out of his cave so you can lure him back into his cave? That makes tons of sense! :smallsigh:

Juntao112
2013-02-05, 01:36 AM
It's something no one would do. Therefore, it is so unexpected that you should do it.

Greenish
2013-02-05, 01:38 AM
Well, to be fair, there's only so much any non-caster can do to a CR appropriate dragon.

Though, I'd like to remind would-be dragon kiters that the slowest wyrmling dragons have flight speed of 100 ft., and it only goes up as they mature.

AsteriskAmp
2013-02-05, 01:41 AM
Well, to be fair, there's only so much any non-caster can do to a CR appropriate dragon.

Though, I'd like to remind would-be dragon kiters that the slowest wyrmling dragons have flight speed of 100 ft., and it only goes up as they mature.Shivering touch?

Greenish
2013-02-05, 01:47 AM
Shivering touch?What of it?

ArcturusV
2013-02-05, 01:48 AM
Nah Greenish, the problem isn't that people forget about Flight necessarily. It's that they presume all monsters are morons. And that they don't have basic pattern recognition and problem solving skills that anything with 14+ Int should have.

AsteriskAmp
2013-02-05, 01:52 AM
What of it?
Upon rereading I notice it says non-casters...

Mystic Muse
2013-02-05, 01:53 AM
Nah Greenish, the problem isn't that people forget about Flight necessarily. It's that they presume all monsters are morons. And that they don't have basic pattern recognition and problem solving skills that anything with 14+ Int should have.

I think some DMs do it more for the fun of the game than because of what the dragon would logically do.

I mean, if only one person in the party can fight one of the most iconic fantasy monsters, where's the fun in that?

tyckspoon
2013-02-05, 01:54 AM
Though, I'd like to remind would-be dragon kiters that the slowest wyrmling dragons have flight speed of 100 ft., and it only goes up as they mature.

Although as their flight speeds go up, so do their sizes, and their maneuverability ratings go down. Older dragons can fly really fast, but they need huge amounts of space to do it in- if they're still lairing in a cave, it's either a really enormous one, or they're traversing it with their land speed until they get to a point that is open enough to take off.

Kelb_Panthera
2013-02-05, 01:57 AM
It's something no one would do. Therefore, it is so unexpected that you should do it.

It leaves you in a cave with a dragon blocking the entrance. It's also completely discarding the element of suprise, though so is setting off the traps instead of avoiding them.

Btw, why would the dragon set off the traps in his own cave? He knows where they are and has no reason not to take his time getting out of the cave since chasing you down on the wing once he gets outside will be trivial.

Lets review.

Deliberately throw away any chance for a suprise attack.Then either
let the casters handle it while the dragon kites the party by strafing with his breath.or put yourself between an annoyed dragon and his hoard inside his own lair.

I like the monk as much as anyone but this is just terrible strategy.

ArcturusV
2013-02-05, 01:59 AM
Yeah. I wasn't talking about taking EVERYONE out. If you're fighting in a lair, you've just neutralized Flight. So anyone can more or less throw in. But there's a huge range between pretending you're an ultra optimized wizard with Int 40... and presuming your monster can be fooled by the same tricks that might work on an Int 2 Dog like faking a throw and making them chase something just because, etc.

Pickford
2013-02-05, 02:06 AM
Least you're throwing out reasons now why your plan might be more relevant. So there's a plus. Though the infinite Shuriken Kiting I don't see working out.

It wouldn't need to be infinite. But the shruikens weigh basically nothing (5 are 1/2 a lb) so if you went in prepared you'd have more than enough. So that's not a disqualifying factor.


Note that I'm not just pulling "Random counters out of a hat" or shaping situations based entirely on the aspect of just countering your plan. I'm doing reasonable things that any DnDverse should account for. Food and Water, for the army example, is a low level cleric spell that 1 in about 250 people can cast if I roughly remember my World Building Demographics as listed in the books.

It's a 3rd level spell (5th level caster required) and feeds three humans (or one horse)/level. So 15 humans or 5 horses for each casting. So any given character capable of casting it could feed (assuming all uses) themselves and 14 friends or 5 of their mounts. You'd need a ratio of 4 level 5 clerics to every 11 cavalry just to feed them. I suspect that is not probable given a 1 in 250 caster ratio for the 'world'. The ratio of casters in an army is going to be significantly worse.


Any army capable of sacking a city would have more than enough people to be including spellcasters (After all, they are infinitely useful, even low level ones).

Just like modern armies are taught to boil water and purify it before drinking, you can expect that a mythical DnDverse army would have spellcasters able to cast "Purify Food and Drink" several times a day.

Modern armies are taught that because of the collective research experience of the past and it works...to a degree. You shouldn't be drinking still water even filtered through charcoal, boiled and dosed with iodine, unless you're desperate. However these aren't modern armies. They aren't even Napoleon invading russia level era. These are an orc horde which would in no way comprehend that. Bacteria are beyond the ken of d&d societies.


Which covers more than enough space at 1 cubic foot/level to cover their logistical needs (8 gallons per casting, which could work for 64 people, even a first level cleric could cast it 4 times a day for roughly 256, covering the demographic spread), and would instantly invalidate your plan.

That does work...but it doesn't stop disease from recurring. So if they purify a well with a corpse in it...they still get sick. If they don't know it's tainted they have no reason to purify, in which case they get sick. Clerics are unlikely to be in the outrider groups, those would be pure warriors. They have to drink, they get infected and the disease incubates. By the time it's noticed, it is too late and you're going to see casualties on a mass scale.

If you want to discuss modern armies:
http://www.pbs.org/wgbh/amex/fever/peopleevents/e_cuba.html
Yellow Fever, for example, reduced the Spanish forces from 230,000 to 55,000 capable of fighting. And that was as recently as 1898. The armies we are talking about would be circa 1200 when armies knew even less about hygiene and how diseases spread.


And is just basic smarts that you'd expect any army to do. Not even like it's a rare splatbook or anything, PHB.

It's smart. Too smart for the characters and people we're discussing. Magic used as a crutch degrades science and research capability.



Not to mention that in a DnDverse you don't take cities by Siege. You take them by force. With the presence of Magic, a simple stone wall is NOT going to hold off an army indefinitely. And that's even IF the city has it. A lot of DnD cities don't have anything nearly as robust as stone wall 20 feet thick and 40 feet high, etc. You can't plan to hold them off for days. Hours maybe. They're going to rush you. It's a given. You're not going to have an army sitting outside your doors for a year.

This is a d&d city, it's walls are reinforced by magic (Read: Immune to spells) you can't take it by force as easily. But in case you're wondering castles have rarely been taken by force, it's almost universally something that happens by starving out defenders.


But the dragon example? You don't have unlimited shuriken, and they do **** damage. A dragon gets poked for that 1d3, it's going to look at you, maybe shoot off a breath weapon. Which is going to hurt for a lot more unless you save.

Actually in the example I was using I would hit the dragon and go around a corner. To even see me the dragon would need to use it's full move (no attack). Rinse and repeat. It's 1d3 damage + str so 5-8 damage assuming no other enhancements/feats. But yeah, it would take 100 rounds of this to kill a Great Wyrm. Sure, tedious, but basically inevitable courtesy of my superior speed.


You run off. The Dragon, having learned you are not a threat to it in the least with your piddly 1d3 damage... sits there. To bait like that you have to prove yourself a danger, or give them a reason to chase just beyond "I exist". Dragons typically aren't stupid. They're going to figure if a lone nutter tries to attack them with a meaningless attack, and run away, it's probably a distraction so someone can steal their horde, or a trap.

In which case I can carry out hit and run attacks with impunity, and kill the dragon. This would take at most 20 rounds against a juvenile. (13HD)


So you come back in. Throw another 1d3. It breath attacks you again. Maybe casts a spell which your evasion won't apply to if you successfully saved, because, again, they are rocking enough INT to be able to realize something isn't working and adapt.

The spells a black dragon can cast aren't exactly something to write home about. But the dragon doesn't get the breath weapon because I'm out of sight when it's his turn and he has to use his turn to even see me. By which time it's too late.


Well, to be fair, there's only so much any non-caster can do to a CR appropriate dragon.

Though, I'd like to remind would-be dragon kiters that the slowest wyrmling dragons have flight speed of 100 ft., and it only goes up as they mature.

It's in a cave, 60' is their max on foot. Which is too slow.

Edit:

Btw, why would the dragon set off the traps in his own cave?

The traps in question were the ones laid prior to luring him out. He doesn't know about these traps because they aren't his.

Further Edit: And that's why you wouldn't bother fighting the dragon outdoors.

ArcturusV
2013-02-05, 02:11 AM
Why would a dragon carve out a lair in such a way that there is obvious cover within... 45' of it's lair? That's what you're talking about to be able to run out, attack, run back. And that's if you have some feat which allows you to attack while moving.

Instead it'd be "You run out. You shuriken. You're standing in the open to be able to hit it. It crushes you on it's turn."

Or "You run out. You're in the open. The dragon stares at you and crushes you before you could throw a shuriken on your second turn and run away."

GenericMook
2013-02-05, 02:13 AM
So would anyone care to explain to me why a Black Dragon that's appropriate for a CR10-12 encounter - which would have 12 Int - stay in a cave with one trapped passage in that it has to walk through?

Anything with a 12 Int that has a natural ability to fly should realize how moronic that is.

Story
2013-02-05, 02:15 AM
So you're plan to defeat the army is ... killing catgirls?

TaiLiu
2013-02-05, 02:17 AM
This is a d&d city, it's walls are reinforced by magic (Read: Immune to spells) you can't take it by force as easily. But in case you're wondering castles have rarely been taken by force, it's almost universally something that happens by starving out defenders.

How? I cannot think of a method to get a wall to be completely immune to spells.



Actually in the example I was using I would hit the dragon and go around a corner. To even see me the dragon would need to use it's full move (no attack). Rinse and repeat. It's 1d3 damage + str so 5-8 damage assuming no other enhancements/feats. But yeah, it would take 100 rounds of this to kill a Great Wyrm. Sure, tedious, but basically inevitable courtesy of my superior speed.

The spells a black dragon can cast aren't exactly something to write home about. But the dragon doesn't get the breath weapon because I'm out of sight when it's his turn and he has to use his turn to even see me. By which time it's too late.

The dragon could use a held action to cast a spell or use an attack.

GenericMook
2013-02-05, 02:21 AM
Oh, and Pickford, I believe you're completely ignoring the fact that the Black Dragon also has spells as a Sorcerer of its level.


Spells
A dragon knows and casts arcane spells as a sorcerer of the level indicated in its variety description, gaining bonus spells for a high Charisma score. Some dragons can also cast spells from the cleric list or cleric domain lists as arcane spells.

Edit: Derp, misread stuff.

Pickford
2013-02-05, 02:22 AM
Why would a dragon carve out a lair in such a way that there is obvious cover within... 45' of it's lair? That's what you're talking about to be able to run out, attack, run back. And that's if you have some feat which allows you to attack while moving.

Instead it'd be "You run out. You shuriken. You're standing in the open to be able to hit it. It crushes you on it's turn."

Or "You run out. You're in the open. The dragon stares at you and crushes you before you could throw a shuriken on your second turn and run away."

You might just as well ask where this dragon installed all these traps.

If we're talking a juvenile the monk with 2 less HD can outmelee it, so the question is moot. If it's bigger than a combined effort isn't unreasonable at level 10 since that's +6 HD.

Oh and they start having SR 17+ as a young adult, so a level 10 mage would have a 30% chance of SR spells flubbing and given the saves a poor chance of any given spell landing. Best shot is Reflex, but even then the beastie only has to roll a 5 to save. So...yeah.



Oh, and Pickford, I believe you're completely ignoring the fact that the Black Dragon also has spells as a Sorcerer of its level.

No I'm not. Juveniles don't have spells, they only get them at Young Adult. But a young Adult is 16 HD whereas the Juvenile is 12 HD.

Given that we're talking a level '10' monk which restricted my options somewhat. A 16 monk could just quivering palm for a 50% chance of killing the juvenile outright.

Greenish
2013-02-05, 02:23 AM
So you're plan to defeat the army is ... killing catgirls?Maybe it was an army of catgirls.

tyckspoon
2013-02-05, 02:25 AM
So would anyone care to explain to me why a Black Dragon that's appropriate for a CR10-12 encounter - which would have 12 Int - stay in a cave with one trapped passage in that it has to walk through?

Anything with a 12 Int that has a natural ability to fly should realize how moronic that is.

Black Dragons in particular are also aquatic creatures; they can breath water naturally and operate just as well in water as they do on land. They often either lair in or build water passages into the approaches to their lairs. The 'run away through the infinite maze of narrow twisty passages' plan is going to hit a significant snag when you have to navigate 100 feet of said narrow twisty passages in pitch black water that the dragon doesn't even notice when it's chasing you, thanks to having a natural Swim speed and Blindsense to tell it where things are there without having to rely on sight.

JaronK
2013-02-05, 02:27 AM
Generally a mage is going to target touch AC against a dragon. Shivering touch is the usual one shot TKO spell for dragons.

But seriously, assuming the dragon has any intelligence at all, it's going to build a lair that lets it use its advantages (flight, strafing with breath weapons, swimming) and not help intruders. Let's at least assume that.

JaronK

Pickford
2013-02-05, 02:44 AM
How? I cannot think of a method to get a wall to be completely immune to spells.

The dragon could use a held action to cast a spell or use an attack.

True, though the dragon would have to know what I was up to to do that. So I'd get at least a couple of hits off. If the dragon doesn't follow me I just wait.

For the walls, there are several possible methods:

1) Permanancied walls of force or Prismatic Walls (Or even both, layered)
2) Make them into stone golems (constructs). Immune to magic.

edit:


Generally a mage is going to target touch AC against a dragon. Shivering touch is the usual one shot TKO spell for dragons.

But seriously, assuming the dragon has any intelligence at all, it's going to build a lair that lets it use its advantages (flight, strafing with breath weapons, swimming) and not help intruders. Let's at least assume that.

JaronK

I guess that's a good reason to kill the mage when they try to cast it while in melee range. Or to immediately fly away when it sees you cast the spell.

Kelb_Panthera
2013-02-05, 02:45 AM
You want reasons for the shuriken plan to fail, how about the dragon's flat-footed AC being high enough that you'll miss almost as often as you hit, if you're throwing them from 10ft away, and from young-adult onward they -all- have DR 5/magic shaving 5 points of your 1d3 weapon?

Combine this with the necessity of getting within 50ft to even be able to hit it with a shuriken, at a noticeably reduced rate, and still needing move speed left and a shot-on-the-run type feat to be able to get out and this plan is utterly worthless.

It's, "move to within 50ft=>throw a shuriken and miss=>duck around the corner that you can't count on existing if you have the right feat=>hope the creature is annoyed or curious enough to follow."

"If dragon follows, lure him out of the cave to where you can no longer attack him at all through traps that you hope he didn't hear you setting up in a place where they'll be painfully obvious to see=>lead him back into the cave where you have no avenue of escape if any of the brilliant steps leading up to this haven't sufficiently weakened the creature."

or

"If the creature ignores you, throw a few more shuikens until you've annoyed it enough to take a readied action to pounce on you when you come back around the corner=>get mauled and eaten by an annoyed dragon while your friends wait outside."

Greenish
2013-02-05, 02:50 AM
2) Make them into stone golems (constructs). Immune to magic.The game being what it is, Immunity to Magic doesn't actually mean the creature is immune to magic. :smallcool:

GenericMook
2013-02-05, 02:51 AM
1) Permanancied walls of force or Prismatic Walls (Or even both, layered)
2) Make them into stone golems (constructs). Immune to magic.

1) Can still be taken down via disintegrate or whatever takes down that specific phase of Prismatic Wall.
2) Then they're golems. Not walls.

And this is gonna cost a ridiculous amount of gold. Only adventurers or ridiculously rich people can afford something mad like this on any sort of fortification.

JaronK
2013-02-05, 02:54 AM
.For the walls, there are several possible methods:

1) Permanancied walls of force or Prismatic Walls (Or even both, layered)
2) Make them into stone golems (constructs). Immune to magic.

Do you have any idea how much that would cost?


I guess that's a good reason to kill the mage when they try to cast it while in melee range. Or to immediately fly away when it sees you cast the spell.

Spectral Hand. 2nd level spell. Now the mage can hit from a safer distance (far beyond the range at which the Monk is useful). The mage isn't getting into melee range. And there's no need to give the dragon time to see the spell being cast.

But you're right of course. The mage is the threat so a dragon would target it. The Monk isn't, because he can't do much more than throw shurikens or die in melee range.

JaronK

Pickford
2013-02-05, 02:57 AM
1) Can still be taken down via disintegrate or whatever takes down that specific phase of Prismatic Wall.
2) Then they're golems. Not walls.

And this is gonna cost a ridiculous amount of gold. Only adventurers or ridiculously rich people can afford something mad like this on any sort of fortification.

You need a Mordekainen's Disjunction (9th level) to outright remove a prismatic wall. Wish doesn't work. So yeah, if you're bringing a 17th level mage for the ride they could break the wall....well they could break 'part' of the wall.

Otherwise the wall blocks all magic unless you cast the correct spells in the correct order.

The wall of force is invisible, so as far as the invading army is concerned there would be quite a few blown spells first. If layered correctly (i.e. building around a wall of force) even if you destroyed the physical wall in front you wouldn't be able to breach the walls themselves.

Golems don't have to have a particular shape.

And yeah, it would be more costly than regular stone walls. But we're talking a city-state or country here, they can afford it if facing an existential threat.


Do you have any idea how much that would cost?

Not off the top of my head. Let's find out shall we?

Wall of Force: 5th level = 50gp per 10-ft. square area (20th level would be 1000gp and make a 100 foot wall that was 20 feet high, a respectable wall.)
Permanancy Wall of Force: 6th level = 60gp per level + 12,500gp
Min caster level 13th for Permanancy of a Wall of Force. (780gp)

So 14,280gp per section. That's cheaper than making a single ring of evasion.

Prismatic wall would be 20,000 + 1600 + 960 = 22,560 per 80x40 wall section.

So both effects would be approximately 36,840gp per wall. Granted, you'd have some ridiculously tough walls. Also that much gold is paltry to a country.

MeaningfulName
2013-02-05, 02:57 AM
Modern armies are taught that because of the collective research experience of the past and it works...to a degree. You shouldn't be drinking still water even filtered through charcoal, boiled and dosed with iodine, unless you're desperate. However these aren't modern armies. They aren't even Napoleon invading russia level era. These are an orc horde which would in no way comprehend that. Bacteria are beyond the ken of d&d societies.
I would like to point out that while medieval societies did have a very limited understanding of disease they had sufficient understanding (primarily through experience) to realize that dead bodies in still water sources meant bad bad things for the drinkers. By the way, you don't even need to drop plague infected bodies into still water to taint it, just let a dead body sit in one for a few days and it's going to be fairly useless as drinking water without significant purification efforts (or magic).

On another note, having played several monks, I have to say, I enjoyed it. But not really for any particular reason based on the class design... or perhaps entirely because of it. I agree entirely that the monk is horribly designed and in the average adventure is going to be outclassed by pretty much everybody else. That said, because I had a crap time fighting legitimately I also had to end up thinking up ridiculous schemes to compensate. Admittedly this is probably also because I had a great DM, and a party who was perfectly content to come up with Rube Goldberg machines in order to kill dragons instead of sticking a big sharp piece of metal into it's brain (or blowing it up with thunder). This discussion rather reminds me of it, though no where near as elaborate or ridiculous.

I bring this up because hey, perhaps the monk serves a fun purpose for the average gamer; it forces them to think up something beyond "punch/fireball/stab move on to next and repeat."

I'm being mostly facetious in case you're wondering.

HMS Invincible
2013-02-05, 03:03 AM
No I'm not trolling, I'm saying that each class is strengths and weaknesses. A wizard would be more at risk of having possessions stolen from their room at night or the like. (Whereas the monk would still function at virtually peak efficiency).

At any rate:
1) Frankly it depends on the level of the dragon. We fight and kill it or could collapse the cave entrance and trap it underground. I'm not sure how class skills are particularly relevant in this scenario. (How were you expecting me to answer this differently?) As a side note, a wizard could only cast summon monster V, which isn't that great. Anyone sending in summoned monsters at 10th level might not even have the monsters 'reach' the dragon (depending on cave depth, at 10th level we're talking only 60 seconds of creature time.) but if they did, all you would have achieved is possibly luring the dragon out of the cave...unless he just stayed home that day.

But...if I had to pick a monk class feature to shine here (at 10th level): I'd go with their increased movement speed. This would still be a team effort, but I'd be the bait. While my teammates setup some traps for the dragon in and around the entrance, I would go inside, alone, to throw a shruiken at it. With my +30' speed I can maintain full distance from the dragon and with improved evasion I would, at worst, only take 1/2 damage from it's breath weapon (but more likely no damage at all). I run out of the cave, tumble through the traps and, after the dragon trips them, participate in a good old fashioned beat down.

2) Sorry why does a Monk care? This is almost polar opposite against the fluff of the monk class. Monk's are in this for themselves, not a bunch of commoners who couldn't protect themselves.

A more interesting adventure (in the same scenario-verse) would be to assassinate the head of the underground resistance on behalf of the tyrannical state's rulers. This would involve gaining the trust of the leader by pretending to fight slavers (dealing non-lethal damage only) and "helping" some "slaves" (read: party members) escape. Then, having gained the trust of the foolish rebels a monk could easily gain audience "unarmed" with the leader for the actual hit.

Actually, this could work even better for a 15th level monk. Challenge the leader to a friendly boxing match. During the match the monk uses quivering palm and, hours later while the leader is meeting with someone else, wills the victim to die. The perfect kill, no one even remotely suspects and blame may come to rest on those who were alone with the rebel leader.

3) Again, why does the monk care? I suppose we could simulate the training of peasants to defend themselves using monk weapons. But then, that's not exactly the best use of a monk class feature. Instead, I send my monk to collect plague infected corpses. Because of Purity of Body the monk is safe, the monk then uses his speed to race to each water source in the direction of the orc army and spoil them with festering corpses. The Orcs, denied a source of fresh water for miles die horribly, disease taking far more of a toll than battle ever does. (Historically accurate AND clever).

As to your query about a commoner...
1) Commoner is slow, doesn't have improved evasion and his saves are awful. He gets eaten when the dragon catches him almost immediately, or turned into goo by the acid breath, or blows all his saves and is impaled on a trap.

2) The commoner can't use quivering palm and would never come close to the melee damage assuming no quivering palm. No flurry, weaker BAB.

3) The commoner dies horribly from the plague. Good job commoner.

Edit: Actually the immunity to normal disease and the 11th level immunity to poison are potentially amazing if you use them right. (i.e. Free use of horrifically dangerous poisons from Complete Scoundrel/Complete Adventurer). Just smear them on a sai and you're good to go, 0 chance of poisoning yourself. Otherwise just use contact poisons smeared on your hands. (Black Lotus Extract deals 3d6 Con damage...good night ...anyone else?) DC 35 too.
Not bad, not very good, but not bad. Very creative, though a little...need a word that describes a good plan in theory but not sure how it pans out in execution. Anyway, onto the critique of your response.
1. This scenario consists of 2 parts, killing a dragon, and getting past various traps. You offered several scenarios, with varying degrees of success. Assuming a monk is good at combat(the others will disagree), you'll contribute well to the fighting unless the dragon starts flying. As for getting past traps, do monks have good AC or some way to detect, detour around, or dispose of traps? Assuming traps only target saves is a risky gamble, since it could target AC or fail a save. Losing your surprise round sucks, but it's not needed. Lastly, you mentioned burying the entrance to trap the dragon. While that denies you treasure, it does solve the encounter. How did the monk help collapse the entrance?
1a. The commoner sends chickens to trigger traps and could help in combat with a charger build. Otherwise, is dead weight. Possibly, he could set chickens as bait, but that leads into air combat quickly without a really good trap.

2. You chose to subvert my hook, and work for the bad guys. Ignoring the potential DM rage, the scenario consists of finding contacts, and not getting caught. Assuming you got the evil guy's permission to work for him (****!), you'd need to set up a false flag which uses non-lethal class feature of monk( easier solution, is to kill the slavers, there's more where they came from, but suddenly you grown a conscience.) You'd get a bonus to a bluff check against the initial deception, but I wouldn't want a monk doing it. (Do monks even get bluff or diplomacy?) The assassination requires bluff vs sense motive, but that is a clever use of a monk feature. It also requires a diplomacy or other social check to convince him to agree. Who the hell agrees to a boxing match with a monk otherwise? I forgot to mention, you need gather information and other perception based skills to find your mark. How many skills is the monk specialized in? Lastly, you forgot to detail how you hid your connections to the BBEG, disguise self? Normally, you're avoiding making a scene to avoid the tyrant's troops but w/e. Overall, monk doesn't do much here, unless he has social skills.
2a. The commoner may have spent a feat and gained diplomacy, bluff or gather information. He may accomplish something here, mostly through diplomacy. Otherwise, deadweight.

3. Clever, using disease and fast movement, but not foolproof. The options here are wide open since it's such a large army. Other classes can assassinate leaders, train armies, fortify the land, or cast magic. Onto your plan, infect the army, retreat to train the peasants and fight the remnants. Armies have scouting parties, can you spot them? Outrun them? After they notice the infection and poisoned water, what happens if they send a team to ambush you and clean the water? Lastly, I know monks are fast, but that fast? Where's your endurance check?
3a. Commoner uses diplomacy or goes home.

Overall, you're pretty creative and clever, but you assume a bit much. Especially trying to argue around rolling a skill check. Sure it could work this time, but are you any better than those players you claim that weren't following the rules since nobody steals spellbooks?

PS Why hate on plot hooks? It's a generic plot hook that comes up in any campaign, it's your responsibility to know why you care. The alternative is to stare at the featureless fields while your DM thinks up another plot hook.